Силы действующие на автомобиль при движении: Силы, действующие на автомобиль

Содержание

Силы действующие на автомобиль при движении. Сборник вопросов и задач по физике — Лукашик В.И Главное

На движущийся автомобиль действует ряд сил, часть из которых направлена по оси движения автомобиля, а часть — под углом к этой оси. Условимся называть первые из этих сил продольными, а вторые боковыми.

Рис. Схема сил действующих на ведущее колесо.
а — состояние неподвижности; б — состояние движения

Продольные силы могут быть направлены как по ходу, так и против хода движения автомобиля. Силы, направленные по ходу движения, являются движущимися и стремятся продолжить движение. Силы, направленные против хода движения, являются силами сопротивления и стремятся остановить автомобиль.

На автомобиль, движущийся по горизонтальному и прямому участку дороги, действуют следующие продольные силы:

  • тяговая сила
  • сила сопротивления воздуха
  • сила сопротивления качению

При движении автомобиля в гору возникает сила сопротивления подъему, а при разгоне автомобиля-сила сопро­тивления разгону (сила инерции).


Тяговая сила

Развиваемый двигателем автомобиля крутящий момент передается на ведущие колеса. В передаче крутящего момента от двигателя к ведущим колесам участвуют механизмы трансмиссии. Крутящий момент на ведущих колесах зависит от крутящего момента двигателя и передаточных чисел коробки передач и главной передачи. В точке касания колес с поверхностью дороги крутящий момент вызывает окружную силу. Противодействие дороги этой окружной силе выражается реактивной силой, передаваемой от дороги на ведущее колесо. Эта сила направлена в сторону движения автомобиля и называется толкающей или тяговой силой. Тяговая сила от колес передается на ведущий мост и далее на раму, заставляя автомобиль двигаться. Величина тяговой силы тем больше, чем больше крутящий момент двигателя и передаточные числа коробки передач и главной передачи. Тяговая сила на ведущих колесах дости­гает наибольшей величины при движении автомобиля на низшей передаче, поэтому низшую передачу используют при трогании с места автомобиля с грузом, при движении автомобиля по бездорожью.

Величина тяговой силы на ведущих колесах автомобиля ограничивается сцеплением шин с поверхностью дороги.

Сила сцепления колес с дорогой

Трение, возника­ющее между ведущими колесами автомобиля и дорогой, называется силой сцепления. Сила сцепления равна произведению коэф­фициента сцепления на сцепной вес, т. е. вес, приходящийся на ведущие колеса автомобиля. Величина коэффициента сцепления шин с дорогой зависит от качества и состояния дорожного покрытия, формы и состояния рисунка протектора шины, давления воздуха в шине.

У легковых автомобилей полный вес рас­пределяется по осям примерно поровну. Поэтому сцепной вес его можно принять равным 50% полного веса. У грузовых автомоби­лей при полной их на­грузке сцепной вес (вес, приходящийся на заднюю ось) составляет примерно 60-70% полного веса.

Величина коэффициента сцепления имеет большое значение для эксплуатации автомобиля и безопасности движения, так как от него зависят проходимость автомобиля, тормозные качества, возможность, пробуксовки и заноса ведущих колес. При незначи­тельном коэффициенте сцепления трогание автомобиля с места со­провождается пробуксовкой, а торможение — скольжением колес. В результате автомобиль иногда не удается тронуть с места, а при торможении происходит резкое увеличение тормозного пути и возникновение заноса.

На асфальтобетонных покрытиях в жаркую погоду на поверх­ность выступает битум, делая дорогу маслянистой и более скольз­кой, что снижает коэффициент сцепления. Особенно сильно снижается коэффициент сцепления при смачивании дороги первым дождем, когда образуется еще не смытая пленка жидкой грязи. Заснежённая или обледенелая дорога особенно опасна в теплую погоду, когда поверхность подтаивает.

При увеличении скорости движения коэффициент сцепления снижается, в особенности на мокрой дороге, так как выступы ри­сунка протектора шины не успевают продавливать пленку влаги.

Исправное состояние рисунка протектора шины имеет большое значение при движении по грунтовым дорогам, снегу, песку, а также по дорогам с твердым покрытием, по покрытым пленкой грязи или воды. Благодаря наличию выступов рисунка опорная площадь уменьшается и, следовательно, возрастает удельное давление на поверхность дороги. При этом легче продавливается грязевая пленка и восстанавливается контакт с дорожным покрытием, а на легком грунте происходит непосредственное зацепление выступов рисунка за грунт.

Повышенное давление воздуха в шине уменьшает ее опорную поверхность, вследствие чего удельное давление возрастает на­столько, что при трогании с места и при торможении может произойти разрушение резины и сцепление колес с дорогой уменьшается.

Таким образом, величина коэффициента сцепления зависит от многих условий и может изменяться в довольно значительных пределах. Так как много дорожно-транспортных происшествий происходит из-за плохого сцепления, то водители должны уметь приблизительно оценивать величину коэффициента сцепления и выбирать скорость движения и приемы управления в соответствии с ним.

Сила сопротивления воздуха

При движении автомобиль преодолевает сопротивление воздуха, которое складывается из нескольких сопротивлений:

  • лобового сопротивле­ния (около 55-60% всего сопротивления воздуха)
  • создаваемого выступающими частями-подножками автобуса или автомобиля, крыльями (12-18%)
  • возникающего при прохождении воздуха через радиатор и подкапотное пространство (10-15%) и др.

Передней частью автомобиля воздух сжимается и раздвигает­ся, в то время как в задней части автомобиля создается разреже­ние, которое вызывает образование завихрений.

Сила сопротивления воздуха зависит от величины лобовой, поверхности автомобиля, его формы, а также от скорости движе­ния. Лобовую площадь грузового автомобиля определяют как произведение колеи (расстояние между шинами) на высоту авто­мобиля. Сила сопротивления воздуха возрастает пропорционально квадрату скорости движения автомобиля (если скорость возра­стает в 2 раза, то сопротивление воздуха увеличивается в 4 раза).

Для улучшения обтекаемости и уменьшения сопротивления воздуха ветровое стекло автомобиля располагают наклонно, а вы­ступающие детали (фары, крылья, ручки дверей) устанавливают заподлицо с внешними очертаниями кузова. У грузовых автомоби­лей можно уменьшить силу сопротивления воздуха, закрыв грузо­вую платформу брезентом, натянутым между крышей кабины и задним бортом.

Сила сопротивления качению

На каждое колесо ав­томобиля постоянно действует вертикальная нагрузка, которая вызывает вертикальную реакцию дороги. При движении автомобиля на него действует сила сопротивления качению, которая возникает вследствие деформации шин и дороги и трения шин о дорогу.

Сила сопротивления качению равна произведению полного веса автомобиля на коэффициент сопротивления качению шин, который зависит от давления воздуха в шинах и качества дорожного покрытия. Вот- некоторые значения коэффициента сопротивления качению шин:

  • для асфальтобетонного покрытия- 0,014-0,020
  • для гравийного покрытия-0,02-0,025
  • для песка-0,1-0,3

Сила сопротивления подъему

Автомобильная дорога состоит из чередующихся между собой подъемов и спусков и редко имеет горизонтальные участки большой длины.

При движении на подъем автомобиль испытывает дополнитель­ное сопротивление, которое зависит от угла наклона дороги к гори­зонту. Сопротивление подъему тем больше, чем больше вес автомобиля и угол наклона дороги. При подъезде к подъему необходимо правильно оценить возможности преодоления подъема.

Если подъем непродолжительный, его преодолевают с разгоном автомобиля перед подъемом. Если подъем продолжительный, его преодолевают на пониженной передаче, переключившись на нее у начала подъема.

При движении автомобиля на спуске сила сопротивления подъему направлена в сторону движения и является движущей силой.

Сила сопротивления разгону

Часть тяговой силы при разгоне затрачивается на ускорение вращающихся масс, главным образом маховика коленчатого вала двигателя и колес автомобиля. Для того чтобы автомобиль начал двигаться с определенной скоростью, ему необходимо преодолеть силу сопротивления разгону, равную произведению массы автомобиля на ускорение. При разгоне автомобиля сила сопротивления разгону направлена в сторону, об­ратную движению. При торможении автомобиля и замедлении его движения эта сила направлена в сторону движения автомобиля. Бывают случаи, когда при резком разгоне груз или пассажиры падают из открытого , с сидений мотоцикла, а при резком торможении пассажиры ударяются о лобовое стекло или о перед­ний борт автомобиля.

Для того чтобы таких случаев не было, необходимо, плавно увеличивая частоту вращения коленчатого вала двигателя, преодолевать силу сопротивления разгону и плавно осу­ществлять торможение автомобиля.

Центр тяжести

На автомобиль, как и на любое другое тело, действует сила тяжести, направленная вертикально вниз. Центром тяжести автомобиля называют такую точку автомобиля, от которой вес автомобиля распределяется равномерно во всех направлениях. У автомобиля центр тяжести располагается между передней и задней осью на высоте около 0,6 м для легковых и 0,7-1,0 м для гру­зовых. Чем ниже расположен центр тяжести, тем устойчивее авто­мобиль против опрокидывания. При загрузке автомобиля грузом центр тяжести поднимается у легковых автомобилей примерно на 0,3-0,4 м, а у грузовых на 0,5 м и более в зависимости от рода груза. При неравномерном укладывании груза центр тяжести может также сместиться вперед, назад или в сторону, при этом будут нарушаться устойчивость автомобиля и легкость управления.

319. Зачем в гололедицу тротуары посыпают песком?
320. Зачем зимой задние колеса некоторых грузовых автомобилей перевязывают цепями?
321. Для чего при спуске воза с горы одно колесо телеги иногда закрепляют так, чтобы оно не вращалось?
322. Зачем на шинах автомашин, колесных тракторов делают глубокий рельефный рисунок (протектор)?
323. Зачем осенью у трамвайных линий, проходящих эколо парков, бульваров и садов, вывешивается предупреждающий знак «Осторожно, листопад!»?

324. Почему после дождя грунтовая дорога скользкая?
325. Почему после дождя опасно съезжать на автомобиле по грунтовой дороге под уклон?


Рис. 79

326. Зачем некоторые мастера смазывают мылом шуруп и винчиванием его в скрепляемые детали?
327. Зачем стапеля, по которым судно спускают в, обильно смазывают?
328. Для чего делается насечка около шляпки гвоздя?
329. Назовите одну-две детали велосипеда, изготовленных с учетом увеличения силы трения скольжения.
330. Какой вид трения возникает при движении каранда-н случаях, указанных на рисунке 78? Куда направлена сила трения по отношению к карандашу в случае а, в случае б-относительно книги?
331. Тележка с грузом движется (рис. 79). Какой вид иия возникает между: а) столом и колесами; б) грузом ккой; в) осями колес и корпусом тележки?
332. Почему кирпичи не скатываются вниз (рис. 80 и 81)? i сила удерживает их в состоянии покоя? Изобразите действующие на кирпичи.
333. Брусок двигают вправо (рис. 82). Куда направлена i трения скольжения по отношению к бруску; относимо поверхности, по которой движется брусок?
334. Лестница у стены занимает положение, изображении рисунке 83. Укажите направление силы трения в месте соприкосновения лестницы со стеной и полом.


Рис. 80


Рис. 81


Рис. 82


Рис. 83


Рис. 84


Рис. 85


Рис. 86

335. Брусок движется равномерно (рис. 84). Куда направлена: а) сила упругости горизонтальной части нити; б) вертикальной; в) сила трения скольжения относительно поверхности стола, относительно бруска; г) чему равна равнодействующая этих сил?
336. Колесо автомобиля буксует (рис. 85). Куда направлена сила трения скольжения между буксующим колесом и дорогой относительно: а) колеса; б) дороги? Куда направлена сила упругости дороги?
337. Книга прижата к вертикальной поверхности (рис. 86). Изобразите графически направления сил тяжести и трения покоя, действующих на книгу.
338. Тележка равномерно дви жется вправо (см. рис. 79). Какая ] сила приводит в движение груз, поставленный на нее? Куда направлена эта сила?
339. На транспортере равномерно движется ящик с грузом (без скольжения). Куда направлена сила трения покоя между лентой транспортера и ящиком, когда ящик: а) поднимается; б) движется горизонтально; в) опускается?


Рис. 87

340. Если автобус равномерно движется по горизонгаль-.iy участку пути, чему равна сила трения покоя?
341. Парашютист, масса которого 70 кг, равномерно опускается. Чему равна сила сопротивления воздуха, действуюют на парашютиста?
342. С помощью динамометра равномерно перемещают г’сок (см. рис. 82). Чему равна сила трения скольжения между бруском и поверхностью стола? (Цена деления динамометра 1 Н.)
343. Зубья пилы разводят в разные стороны от плоскости пилы. На рисунке 87 показаны пропилы, сделанные неразведенной и разведенной пилами. Какой пилой труднее пилить: разведенной или неразведенной? Почему?
344. Приведите примеры, когда трение приносит пользу и когда вред.

Предыдущая 1 .. 10 > .. >> Следующая
333. Брусок двигают вправо (рис. 82). Куда направлена сила трения скольжения?
334. Лестница у стены занимает положение, изображенное на рисунке 83. Укажите направление силы трения в местах соприкосновения лестницы со стеной и полом.
335. Брусок движется равномерно (рис. 84). Куда направлены сила упругости нити и сила трения скольжения, возникающая при движении бруска по поверхности стола? Чему равна равнодействующая этих сил?
336. Колесо автомобиля буксует (рис. 85). Куда направлена сила трения скольжения между буксующим колесом и дорогой? сила трения покоя (упругости дороги)?
Рис. 86
Рис. 87
837. Книга прижата к вертикальной поверхности (рис. 86). Изобразите графически направления сил тяжести и трения покоя, действующих на книгу.
338. Тележка равномерно движется (см. рис. 79). Какая сила приводит в движение груз, лежащий на тележке? Куда она направлена?
339. На транспортере движется ящик с грузом (без скольжения). Куда направлена сила трения покоя между лентой транспортера и ящиком?
340. Если автобус равномерно движется по горизонтальному пути, чему равна сила трения покоя?
341. Парашютист, масса которого 70 кг, равномерно движется. Чему равна сила сопротивления воздуха, действующая на парашют?
342. С помощью динамометра равномерно перемещают брусок (см. рис. 82). Чему равна сила трения скольжения между бруском и поверхностью стола? (Цена деления динамометра 1 Н.)
343. Зубья пилы разводят в разные стороны от плоскости пилы. На рисунке 87 показаны пропилы, сделанные неразведенной и разведенной пилами. Какой пилой труднее пилить: разведенной или неразведенной? Почему?
344. Приведите примеры, когда трение полезно и когда оно приносит вред.
17. ДАВЛЕНИЕ1
345. Два тела равного веса поставлены на стол так, как показано на рисунке 88 (слева). Одинаковое ли давление они производят на стол? Если эти тела поставить на чашки весов, то нарушится ли равновесие весов?
346. Одинаковое ли давление оказываем мы на карандаш, затачивая его тупым и острым ножом, если прилагаемое нами усилие одно и то же?
1 При расчетах принимать g=10 Н/кг.
37
347. Перемещая одинаковый груз (рис. 89), мальчики в первом случае прикладывают большую силу, чем во втором. Почему? В каком случае давление груза на пол больше? Почему?
348. Зачем у лопаты верхний край, на который надавливают ногой, изогнут?
349. Для чего у косилки, соломорезки и других сельскохозяйственных машин режущие части должны быть остро отточены?
350. Зачем для проезда по болотистым местам делают настил из хвороста, бревен или досок?
351. Когда скрепляют болтом деревянные бруски, под гайку и головку болта подкладывают широкие металлические плоские кольца — шайбы (рис. 90). Для чего это делают?
352. Для чего при вытаскивании гвоздей из доски подкладывают под клещи железную полоску или дощечку?
353. Объясните назначение наперстка, надеваемого на палец при шитье иглой.
354. В одних случаях давление стараются уменьшить, а в других — увеличить. Приведите примеры, где в технике или в быту уменьшают, а где увеличивают давление.
355. На рисунке 91 изображен кирпич в трех положениях. При каком положении кирпича давление на доску будет наименьшим? наибольшим?
Рис. 89
Рис. 91
Рис. 90
38
3
Рис. 92
Рис. 93
356. Одинаковое ли давление производят на стол кирпичи, расположенные так, как показано на рисунке 92?
357. Два кирпича поставлены друг на друга так, как показано на рисунке 93. Одинаковы ли силы, действующие на опору, и давление в обоих случаях?
358. Розетки прессуют из специальной массы (барка-литовой), действуя на нее силой 37,5 кН. Площадь розетки 0,0075 м2. Под каким давлением находится розетка?
359. Площадь дна кастрюли равна 1300 см2. Вычислите, на сколько увеличится давление кастрюли на стол, если в нее налить воду объемом 3,9 л.
360. Какое давление на пол производит мальчик, масса которого 48 кг, а площадь подошв его обуви 320 см2?
361. Спортсмен, масса которого 78 кг, стоит на лыжах. Длина каждой лыжи 1,95 м, ширина 8 см. Какое давление оказывает спортсмен на снег?
362. Токарный станок массой 300 кг опирается на фундамент четырьмя ножками. Определите давление станка на фундамент, если площадь каждой ножки 50 см2.
363. Лед выдерживает давление 90 кПа. Пройдет ли по этому льду трактор массой 5,4 т, если он опирается на гусеницы общей площадью 1,5 м2?
364. Двухосный прицеп с грузом имеет массу 2,5 т. Определите давление, оказываемое прицепом на дорогу, если площадь соприкосновения каждого колеса с дорогой равна 125 см2.
365. На железнодорожную двухосную платформу поставили артиллерийское орудие массой 5,5 т. На сколько увеличилось давление платформы на рельсы, если площадь соприкосновения колеса с рельсом 5 см2?
366. Вычислите давление, производимое на рельсы четырехосным груженым вагоном массой 32 т, если площадь соприкосновения колеса с рельсом 4 см2.
39
Рис. 95
Рис. 96
367. Какое давление оказывает на грунт гранитная колонна, объем которой 6 м3, если площадь основания ее 1,5 м*?
368. Можете ли вы гвоздем оказать давление 105 кПа? Рассчитайте, какую силу для этого надо приложить к головке гвоздя, если площадь острия гвоздя равна 0,1 мм2.

400. Зачем в гололедицу тротуары посыпают песком?
Для того, чтобы увеличить коэффициент трения. В этом случае вероятность поскользнуться и упасть будет меньше.

401. Зачем зимой задние колеса некоторых грузовых автомобилей перевязывают цепями?
Для того, чтобы увеличить коэффициент трения и тем самым практически не допустить проскальзывания между колесами автомобиля и обледенелым участком полотна дороги.

402. Для чего при спуске воза с горы одно колесо телеги иногда закрепляют так, чтобы оно не вращалось?
Чтобы увеличить трение между телегой и дорогой. В этом случае скорость телеги будет не очень, большой, зато безопасной для спуска.

403. Зачем на шинах автомашин, колесных тракторов делают глубокий рельефный рисунок (протектор)?
Для увеличения коэффициента трения между колесами и дорогой. В этом случае сцепление с землей будет более эффективным.

404. Зачем осенью у трамвайных линий, проходящих около парков, бульваров и садов, вывешивается предупреждающий знак «Осторожно, листопад!»?
Сухие листья уменьшают сцепление колес трамвая с рельсами, вследствие чего может возникнуть пробуксовка колес, тормозной путь трамвая также увеличится.

405. Почему после дождя грунтовая дорога скользкая?
Вода на поверхности земли является смазкой и поэтому уменьшает коэффициент трения.

406. Почему после дождя опасно съезжать на автомобиле по грунтовой дороге под уклон?
Потому что вода на поверхности дороги уменьшает коэффициент трения.

407. Зачем некоторые мастера смазывают мылом шуруп перед ввинчиванием его в скрепляемые детали?
Мыло служит смазкой и уменьшает коэффициент трения. В этом случае процесс ввинчивания шурупа будет более легким.

408. Зачем стапеля, по которым судно спускают в воду, обильно смазывают?
Для того, чтобы уменьшить коэффициент трения между спускаемым судном и стапелями, и тем самым облегчить процесс спуска.

409. Для чего делается насечка около шляпки гвоздя?
Для увеличения коэффициента трения. В этом случае молоток будет меньше соскальзывать со шляпки гвоздя.

410. Назовите одну-две детали велосипеда, изготовленные с учетом увеличения силы трения скольжения.
Резиновая покрышка, тормозные колодки.

411. Какие силы трения возникают при движении карандаша в случаях, указанных на рисунке 93, а, б? Куда направлена сила трения, действующая на карандаш, относительно оси карандаша в обоих случаях?
а) сила трения скольжения; она направлена вдоль оси карандаша в противоположную сторону его движения,
б) сила трения качения; она направлена перпендикулярно к оси карандаша в противоположную сторону его движения.

412. Тележка с грузом движется (рис. 94). Какой вид трения возникает между: а) столом и колесами; б) грузом и тележкой; в) осями колес и корпусом тележки?
а) сила трения качения;
б) сила трения покоя, если груз покоится относительно тележки, или сила трения скольжения, если груз движется;
в) сила трения скольжения.

413. Почему кирпичи не соскальзывают вниз (рис. 95 и 96)? Какая сила удерживает их в состоянии покоя? Изобразите силы, действующие на кирпичи. 

414. Брусок двигают вправо (рис. 97). Куда направлена сила трения скольжения по отношению к бруску; относительно поверхности, по которой движется брусок?
Относительно бруска сила трения скольжения направлена влево (против направления движения). Относительно поверхности, по которой движется брусок, сила трения направлена вправо (по направлению движения).

415. Лестница у стены занимает положение, изображенное на рисунке 98. Укажите направление силы трения в местах соприкосновения лестницы со стеной и полом.

416. Брусок движется равномерно (рис. 99). Куда направлена: а) сила упругости горизонтальной части нити; б) вертикальной части нити; в) сила трения скольжения, действующая на поверхность стола, на брусок? Чему равна равнодействующая этих сил?

417. Колесо автомобиля буксует (рис. 100). Куда направлена сила трения скольжения между буксующим колесом и дорогой, которая действует: а) на колесо; б) на дорогу? Куда направлена сила упругости дороги?

418. Книга прижата к вертикальной поверхности (рис. 101). Изобразите графически направления сил тяжести и трения покоя, действующих на книгу.

419. Тележка равномерно движется вправо (см. рис. 94). Какая сила приводит в движение груз, поставленный на нее? Чему равна эта сила при равномерном движении?
Груз, лежащий на тележке, приводится в движение силой трения покоя, направленной вправо. При равномерном движении те¬лежки эта сила равна нулю.

420. На транспортере равномерно движется ящик с грузом (без скольжения). Куда направлена сила трения покоя между лентой транспортера и ящиком, когда ящик: а) поднимается; б) движется горизонтально; в) опускается?
а) вверх вдоль транспортера; б) она равна нулю; в) вверх вдоль транспортера.

421. Равна ли сила тяги силе трения, если автобус движется без скольжения равномерно: 1) по горизонтальному пути; 2) вверх по наклонному участку пути?
Если автобус движется равномерно по горизонтальному участку пути, то сила трения покоя равна силе тяги за вычетом силы сопротивления воздуха.

422. Парашютист, масса которого 70 кг, равномерно опускается. Чему равна сила сопротивления воздуха, действующая на парашютиста?

423. С помощью динамометра равномерно перемещают брусок (см. рис. 97). Чему равна сила трения скольжения между бруском и поверхностью стола? (Цена деления динамометра 1 Н.)
При равномерном движении бруска сила трения скольжения между бруском и поверхностью стола равна силе упругости пружины динамометра. Поэтому в этом случае динамометр нам показывает значение силы трения скольжения. Согласно рис. 97 она равна 4Н.

424. Зубья пилы разводят в разные стороны от плоскости пилы. На рисунке 102 показаны пропилы, сделанные неразведенной и разведенной пилами. Какой пилой труднее пилить? Почему?
Труднее пилить неразведенной пилой, так как в этом случае боковые поверхности пилы плотнее соприкасаются с деревом и между ними возникает большая сила трения.

425. Приведите примеры, когда трение приносит пользу и когда вред.
Трение приносит пользу при ходьбе, беге, езде на транспорте, движении грузов на транспортере. Трение приносит вред в трущихся деталях различных механизмов, где нежелательно стирание поверхностей.

426. На уроке физкультуры мальчик равномерно скользит вниз по канату. Под действием каких сил осуществляется это движение?
Под действием силы тяжести и силы трения скольжения.

427. Судно буксирует три баржи, соединенные последовательно одна за другой. Сила сопротивления воды для первой баржи 9000 Н, для второй 7000 Н, для третьей 6000 Н. Сопротивление воды для самого судна 11 кН. Определите силу тяги, развиваемую судном при буксировке этих барж, считая, что баржи движутся равномерно.

428. На движущийся автомобиль в горизонтальном направлении действуют сила тяги двигателя 1,25 кН, сила трения 600 Н и сила сопротивления воздуха 450 Н. Чему равна равнодействующая этих сил?

429. Можно ли однозначно утверждать, что приращение силы сопротивления AF равно 3 мН, если скорость тела, движущегося в некоторой среде с коэффициентом сопротивления 0,01, увеличилась на 0,3 м/с?
Однозначно утверждать так нельзя, так как сила сопротивления в вязких средах задается неоднозначно. При малых скоростях движения она пропорциональна скорости, при больших — квадрату скорости.

430. Троллейбус трогается с места и в течение 30 с приобретает импульс 15 104 кг-м/с. Определите силу сопротивления движению, если развиваемая троллейбусом сила тяги равна 15 кН.

431. На автомобиль массой 103 кг во время движения действует сила сопротивления, равная 10% от его веса. Чему должна быть равна сила тяги, развиваемая автомобилем, чтобы он двигался с постоянным ускорением 2 м/с2?

434. Велосипедист, ехавший со скоростью 11 м/с, резко затормозил. Коэффициент трения скольжения шин о сухой асфальт равен 0,7. Определите ускорение велосипедиста при торможении; время торможения; тормозной путь велосипедиста.

435. Какую силу надо приложить в горизонтальном направлении к вагону массой 16 т, чтобы уменьшить его скорость на 0,6 м/с за 10 с; за 1 с? Коэффициент трения равен 0,05.

436. С какой скоростью сможет ехать по горизонтальной плоскости мотоциклист, описывая дугу радиусом 83 м, если коэффициент трения резины о почву равен 0,4?

Всем механикам с юности памятна картинка со схемой движения автомобиля по кривой, когда его внешние колеса проходят больший путь, чем внутренние. С ее помощью во многих учебниках для водителей разъясняются назначение и принцип действия дифференциала. Часто все сводится к тому, что дифференциал позволяет ведущим колесам вращаться с различными скоростями и, таким образом, обеспечивает нормальное движение автомобиля на поворотах.

Такие разъяснения не то чтобы совсем неправильны, но слишком упрощены и сути работы дифференциала не раскрывают. Конечно, в серьезных книгах все изложено правильно. Там сказано, что назначение межколесного дифференциала на автомобиле состоит в распределении крутящего момента строго поровну между ведущими колесами одного моста, а межмостового дифференциала – в распределении крутящего момента между ведущими мостами, — поровну или в оптимальной пропорции (несимметричный дифференциал).

«Дифференциал – это механизм, у которого ведущие колеса вращаются независимо друг от дружки».

Строго говоря, вращаются они «зависимо», ну да ладно, — что-то похожее на правду сказано, а об остальном ни слова, чтобы не забивать голову людям без специальной подготовки.

Зеленин С.Ф., Молоков В.А. Учебник по устройству автомобиля, М., «Русьавтокнига», 2000 г., 80 с. Тираж 15000 экз.

Цитата из этой книги:

« Дифференциал предназначен для распределения крутящего момента между полуосями ведущих колес при повороте автомобиля и при движении по неровностям дороги. Дифференциал позволяет колесам вращаться с разной угловой скоростью и проходить неодинаковый путь без проскальзывания относительно покрытия дороги.

Иными словами 100% крутящего момента, который приходит на дифференциал, могут распределяться между ведущими колесами как 50 х 50, так и в другой пропорции (например, 60 х 40). К сожалению, пропорция может быть и 100 х 0. Это означает, что одно из колес стоит на месте (в яме), а другое в это время буксует (по сырой земле, глине, снегу).

Что поделаешь! Ничто не бывает абсолютно правильным и идеальным, зато данная конструкция позволяет автомобилю поворачивать без заноса, а водителю не менять каждый день напрочь изношенные шины.

Рис. 38 Главная передача с дифференциалом

1 — полуоси; 2 — ведомая шестерня; 3 — ведущая шестерня; 4 — шестерни полуосей; 5 — шестерни-сателиты

Это уже не упрощение, а просто введение в заблуждение читателей. Здесь, кроме второго предложения и иллюстрации, все неправда (в первом предложении нужно вставить слово «поровну», а точку поставить после слова «колес» и т. д.).

Только однажды в учебнике для профтехобразования мне довелось встретить правильное и при этом простое и наглядное разъяснение сути работы дифференциала. Было это давно и помню только, что это был учебник для водителей зерновых комбайнов.

Там читателю предлагалось вообразить, что две полуосевые конические шестерни «развернуты» в две зубчатые рейки, эти рейки лежат на воображаемом столе, а между ними помещен сателлит в виде прямозубой шестерни. Выглядит это примерно так:

Объяснение сути работы дифференциала основано на его конструкции и на третьем законе Ньютона, который гласит: сила действия равна по модулю и противоположна по направлению силе противодействия. На следующем рисунке показано силовое взаимодействие сателлита с рейками, когда движущая сила Д приложена к оси сателлита и этот сателлит толкает обе рейки по столу, причем силы сопротивления движению левой и правой реек С лев и С прав одинаковы (силы трения реек о поверхность воображаемого стола) и каждая из них равна половине общей силы сопротивления С. Силы со стороны сателлита передаются на рейки в точках зацепления зубьев сателлита с зубьями реек. Благодаря равенству сил сопротивления движению С лев и С прав, равны между собой и движущие силы на зубьях сателлита, каждая из которых равна половине движущей силы Д. Поскольку равные силы приложены к двум зубьям сателлита, находящимся на равных расстояниях от его оси, сателлит находится в равновесии и не вращается. Поэтому все три детали движутся прямолинейно в одну сторону и с равными скоростями, а именно с той скоростью, с какой движется ось сателлита и которая задана двигателем.

Эта ситуация соответствует установившемуся движению автомобиля по дороге с хорошим сцеплением с дорогой.

Теперь представим, что при своем движении по столу, левая рейка «наехала» на пятно масла. При этом сила сопротивления ее движению (сила трения о стол) уменьшилась, а сила сопротивления движению правой рейки осталась прежней. На какой-то момент равновесие сил на зубьях сателлита нарушается: нагрузка на левый его зуб становится меньше нагрузки, действующей на его правый зуб. Иначе говоря, сателлиту стало легче толкать левую рейку, чем правую. Поэтому он начинает вращаться по часовой стрелке, как это показано на следующем рисунке.

Благодаря вращению сателлита движение правой рейки замедляется, а левая рейка наоборот ускоряется. Затем правая рейка полностью останавливается, а сателлит продолжает вращаться. Его ось продолжает двигаться с той же скоростью, что и прежде, так как эта скорость задана двигателем. Но поскольку правая рейка стоит, вращающийся сателлит обкатывается по ней. В момент, показанный на рисунке правый зуб сателлита стоит на месте, так как «упирается» в зуб неподвижной рейки. Но противоположный, левый зуб сателлита движется в два раза быстрее, чем ось самого сателлита. Все это соответствует ситуации, когда одно из ведущих колес медленно движущегося автомобиля наезжает, например, на обширное пятно льда, а второе остается на сухом покрытии с хорошим сцеплением. То есть машина останавливается и колесо, находящееся на льду, буксует, вращаясь в два раза быстрее, чем прежде, когда оба колеса катились с одинаковой скоростью.

Строго говоря, о нарушении равновесия сил на зубьях сателлита выше сказано некорректно и только потому, что, как мне кажется, так проще понять происходящее. На самом деле равновесие сил сохраняется всегда, только для его рассмотрения нужно еще учитывать силы, вызывающие ускорение левой рейки и замедление правой. Эти не рассматриваемые нами силы, исчезают с момента полной остановки правой рейки. В этот же момент удвоенная скорость движения левой рейки становится постоянной. И тогда ситуация полностью соответствует следующему рисунку.

Здесь равновесие сил восстановилось, точнее, — исчезли динамические силовые составляющие (те, что вызывали ускорение одной рейки и замедление другой). Правая рейка стоит, сателлит вращается, а левая рейка движется равномерно с удвоенной скоростью. Очень важно отметить что, равновесие сил перешло на новый уровень. Теперь равные силы на левом и правом зубьях сателлита стали существенно меньше прежних. В силу третьего закона Ньютона эти силы не могут превысить движущую силу, которую можно приложить к рейке, находящейся на пятне масла, или к колесу, находящемуся на пятне льда. Иными словами, если одно колесо стоит на сухой дороге, а противоположное буксует на льду или в грязи, это вовсе не означает, что 100% крутящего момента передается от двигателя на буксующее колесо, как сказано в упомянутой выше книге. Этот момент всегда и во всех условиях делится дифференциалом поровну между колесами, но он не может быть больше, чем позволяет сцепление одного из колес с дорогой, причем именно того колеса, у которого это сцепление меньше.

Только если в этих условиях заблокировать дифференциал, то есть выключить его из работы, тем или иным способом жестко соединив между собой полуоси, можно передать на колесо, стоящее на сухой дороге, подавляющую часть крутящего момента, который может развить двигатель. При этом буксование прекратится, оба колеса будут вращаться с одинаковой скоростью, но подавляющую часть суммарной силы тяги будет обеспечивать только одно из этих колес.

Мне кажется, что с помощью модели с зубчатыми рейками можно наглядно объяснить и все прочие режимы работы межколесного дифференциала. Например, ситуацию, иногда возникающую при торможении двигателем. Представим, что автомобиль движется под уклон на сухой дороге с пятнами льда. Водитель тормозит двигателем. В этом случае движущая сила, это сила инерции массы машины. А сила сопротивления движению, это сила, приложенная к осям сателлитов дифференциала со стороны двигателя. Одно из колес наезжает на пятно льда. Сила сцепления этого колеса с дорогой резко уменьшается, и оно начинает вращаться в обратную сторону. Здесь происходит то же самое, что произойдет с рейками если ось сателлита сделать неподвижной, но оставить ему свободу вращения вокруг этой оси, то есть имитировать ситуацию, когда ось сателлита тормозится или удерживается двигателем. Если теперь двинуть вперед одну из зубчатых реек, то сателлит начнет вращаться и заставит вторую рейку двигаться назад. Здесь рейка, движимая вперед, соответствует колесу на сухой дороге, а рейка, движущаяся назад, — колесу, находящемуся на льду и вращающемуся в обратную сторону. На мой взгляд, вращение буксующего колеса в обратную сторону очень наглядно демонстрирует «стремление» дифференциала выполнить свое предназначение и выровнять силы на двух колесах ведущего моста. В данном случае это силы торможения. Благодаря их выравниваю исключается или сильно снижается вероятность заноса автомобиля при таком режиме торможения.

Можно рассматривать еще многие ситуации, возникающие при работе дифференциала. Но полагаю, что и сказанного достаточно, чтобы убедиться: — межколесный дифференциал всегда делит получаемый от двигателя крутящий момент поровну между двумя колесами одного ведущего моста.

А теперь вернемся к упомянутой в самом начале картинке с автомобилем, движущемся по кривой. Если автомобиль заднеприводной, то получающие одинаковый крутящий момент два задних колеса преобразуют эти крутящие моменты в две одинаковые силы тяги (если шины колес имеют одинаковый диаметр, одинаковое давление накачки и несут одинаковые части веса автомобиля). А две одинаковые силы тяги стремятся толкать автомобиль по прямой. Именно поэтому, водителю при прохождении поворота приходится твердо удерживать рулевое колесо. Строго говоря, дифференциал на таком автомобиле не столько помогает, сколько мешает прохождению поворота. Зато он прямо способствует устойчивости движения по прямой (вместе с углами установки передних колес).

У переднеприводного автомобиля ситуация несколько иная. Здесь силы тяги также одинаковы на двух колесах, но они «поворачиваются» вместе с поворачиваемыми колесами. Поэтому, например, переднеприводной машине легче выйти из глубокой скользкой колеи: повернутые передние ведущие колеса активно тянут куда нужно. А у заднеприводного, задние ведущие колеса активно толкают машину вдоль колеи.

Здесь рассмотрена лишь малая часть того, что следовало бы водителям знать о работе дифференциала и на это потребовалось много слов и картинок. Так может быть правы те, кто ограничивается пресловутой картинкой с разным пробегом у разных колес на повороте? Может быть. Но полагаю, что следует, если и не вдаваться в пространные разъяснения, то хотя бы просто написать, для чего действительно предназначен этот механизм. А кто захочет дойти до сути, найдет, где об этом почитать. И уж совсем ни к чему пропагандировать собственное неверное понимание этой сути.

Определение сил, действующих на автомобиль при его движении и торможении

3 Определение сил, действующих на автомобиль при его движении и торможении

Цель работы:

— Научиться определять величину сил, действующих на автомобиль при его движении.

— Научиться определять величину сил, действующих на автомобиль при его торможении.

— Научится определять основные измерители тормозных качеств автомобиля.

— Научится пользоваться расчетными таблицами.

3.1 Ход работы

3.1.1 Данные для расчета

Определить силы действующие на автомобиль при его движении и торможении, если вес автомобиля Ga =       т; имеет задний привод и движется со скоростью           км/ч в гору; угол уклона g=      ; база автомобиля L =     м; максимальная высота автомобиля Н =      м.

3.1.2 Расчетная база

3.1.3 Тяговая сила, действующая на автомобиль.

Fk = G2 · ƒ =                                                                                                                Н

где G2 = 1/3 Gа =          Н — вес автомобиля, приходящийся на ведущие колёса ƒ = 0,5 . .. 0,8 — коэффициент сцепления

3.1.4 Сила сопротивления качению.

Ff = Ga· f =                                                                                                                                          H

где f = 0,018 — коэффициент сопротивлению качению

3.1.5 Лобовая площадь автомобиля.

А = В1 · Н =                                                                         м2

где В1 =     м — колея колёс,

Н =       м — наибольшая высота автомобиля.

3.1.6 Сила сопротивления воздуха.

Fw = Rw · А · Va2 =                                                                        Н

где Rw =      Нс2 — коэффициент обтекаемости автомобиля,

Va =        м/с — скорость автомобиля в м/с

3.1.7 Сила сопротивления подъему

Fh = Ga · jg =                                                                                        Н

где jg = tgag =             — уклон дороги

3. 1.8 Сила инерции

Равна нулю, т.к. автомобиль движется равномерно.

3.1.9 Нормальная реакция, действующая на задние колёса автомобиля

Х2 = Ga · a cosαg/L – f · ha =                                                                                H

где cosαg = cos    0 =

f =      — коэффициент сцепления с дорогой

ha =      м — высота центра массhg

а = 1 / 3  L =          м , где L =        м — база автомобиля

3.1.10 Нормальная реакция, действующая на передние колёса автомобиля.

Х2 = Ga · a cosαg  =                                                    H

3.1.11 Нормальные реакции (в статическом состоянии на горизонтальной плоскости)

X1| = Gab/L =                                                                                                        H

 X1| = Gaa/L =                                                                                                       H

3. 1.12 Коэффициенты продольного распределения нагрузки

m1 = x1/ x1| =                                         

m2 = x2/ x2| =                                         

Основные измерители движения автомобиля при разгоне.

3.1.13 Задаёмся коэффициентом суммарного сопротивления Ψ =         ; принять динамический фактор D =

3.1.14 Коэффициент учёта влияния вращающихся масс

 SBp = 1 + 0,007ik2 =                                   

где ik = 4 — передаточное число коробки передач

3.1.15 Ускорение автомобиля

Ja =   g =                                                                                           м/с2

3.1.16 Время разгона

tp = Vt/1,8 · ja =                                                                                                      c

3. 1.17 Путь разгона

Sp = Vср · tp/1,8 =                                                                                                                                                        м

где Vср  = Vt+ V0/2 =          м/с – средняя скорость автомобиля при разгоне

Силы и реакции, действующие на автомобиль при его торможении

3.1.18 Максимальная тормозная сила

Fтоp = f · Ga=                                                                                                                                                                  Н

3.1.19 Нормальная реакция, действующие на передние колёса.

X 1тор =  Gaa-Fmophg/L=                                                                                   H

3. 1.20 Нормальная реакция, действующие на задние колёса.

X2тор =  Gab-Fmophg/L=                                                                                   H

Основные измерители тормозных качеств автомобиля (рассматриваем процесс торможения при максимальном использовании тормозной силы на горизонтальной дороге, пренебрегая сопротивлением воздуха.

3.1.21 Ускорение при торможении

jтор = f · g =                                                                                                           м/с2

3.1.21 Путь торможения

Sтор = Va2/254f =                                                                                                     м

где Va — 90 км/ч — скорость автомобиля

3. 1.23 Время торможения

tтор = Va/f · g =                                                                                                          с

3.1.24 Дистанция безопасности

Д.Б. = Sтоp + Sр.в. + Sп =

где Sтоp =       м — величина тормозного пути;

Sр.в. =         м — путь пройденный автомобилем за время реакции водителя;

Sп =         м — путь пройденный автомобилем за время срабатывания привода.

3.2 протокол выполнения работы

Таблица 3.2.1 — Силы, действующие на автомобиль при его движении

Таблица 3.2.2 – Силы действующие на автомобиль при его торможении

Таблица 3.2.3 – Основные измерительные качества автомобиля

Вывод:

Контрольные вопросы:

— Какие силы действуют на автомобиль при его движении?

— Что такое дистанция безопасности?

— Как определять ускорение, время, путь при его движении автомобиля?

— Как определить суммарный коэффициент сопротивления дороги?

— Как определить динамический фактор автомобиля?

— Как определить нормальные реакции на колесах автомобиля при его движении и торможении?

— Как определить коэффициенты продольного распределения нагрузки?

Куда направлена сила трения колеса.

Силы действующие на автомобиль при движении

Задача по физике — 5700

2017-12-15
Как направлена сила трения, действующая на ведущие колеса автомобиля, при разгоне (а), торможении (б), повороте (в)? Равна ли эта сила своему максимальному значению $\mu N$ ($\mu$ — коэффициент трения, $N$ — сила реакции полотна дороги), и если да, то в каких ситуациях? А в каких ситуациях нет? Хорошо это, или плохо, если сила трения достигает своего максимального значения? Почему? Какой автомобиль может развивать на дороге большую мощность — передне- или заднеприводный — при одинаковой мощности мотора и почему? Считать, что масса автомобиля распределена равномерно, и его центр тяжести находится посередине.


Решение:

Обсудим сначала вопрос о роли силы трения в движении машины. Представим себе, что водитель машины, стоящей на гладком-гладком льду (сила трения между колесами и льдом отсутствует), нажимает на педаль газа. Что будет происходить? Ясно, что машина ехать не будет: колеса будут вращаться, но будут пробуксовывать относительно льда — ведь трения-то нет. Причем это будет происходить независимо от мощности двигателя. А это значит, что для того, чтобы мощность двигателя использовать, нужно трение — без него машина не поедет.

Что же происходит, когда сила трения есть. Пусть сначала она очень маленькая, а водитель стоящей машины снова нажимает на педаль газа? Колеса (речь сейчас идет о ведущих колесах автомобиля, допустим это передние колеса) проскальзывают относительно поверхности (трение — маленькое), вращаясь так, как показано на рисунке, но при этом возникает сила трения, действующая со стороны дороги на колеса, направленная вперед по ходу движения машины. Она и толкает машину вперед.


Если сила трения большая, то при плавном нажатии на педаль газа колеса начинают вращаться, и как бы отталкиваются от шероховатостей дороги, используя силу трения, которая направлена вперед. При этом колеса не проскальзывают, а катятся по дороге, так, что нижняя точка колеса не перемещается относительно полотна. Иногда и при большом трении колеса пробуксовывают. Наверняка, вы сталкивались с ситуацией, когда какой-нибудь «сумасшедший водитель» так трогается при включении зеленого сигнала светофора, что колеса «визжат», а на дороге остается черный след из-за скольжения резины по асфальту. Итак, в экстренной ситуации (при резком торможении или трогании с побуксовкой) колеса скользят относительно дороги, в обычных случаях (когда на дороге не остается черного следа от стирающихся покрышек) колесо не скользит, а только катится по дороге.

Итак, если машина едет равномерно, то колеса не скользят по дороге, а катятся по ней так, что нижняя точка колеса покоится (а не проскальзывает) относительно дороги. Как в этом случае направлена сила трения? Сказать, что противоположно скорости машины — неверно, ведь говоря так про силу трения, подразумевают случай скольжения тела относительно поверхности, а сейчас у нас скольжения колес относительно дороги нет. Сила трения в этом случае может быть направлена как угодно, и мы сами определяем ее направление. И вот как это происходит.


Представим себе, что нет никаких препятствующих движению машины факторов. Тогда машина движется по инерции, колеса вращаются по инерции, причем угловая скорость вращения колес связана со скоростью движения машины. Установим эту связь. Пусть колесо движется со скоростью $v$ и вращается так, что нижняя точка колеса не проскальзывает относительно дороги. Перейдем в систему отсчета, связанную с центром колеса. В ней колесо как целое не движется, а только вращается, а земля движется назад со скоростью $v$. Но поскольку колесо не проскальзывает относительно земли, то его нижняя точка имеет такую же скорость как земля. А значит, и все точки поверхности колеса вращаются относительно центра со скоростью $v$ и, следовательно, имеют угловую скорость $\omega = v / R$, где R — радиус колеса. Переходя теперь назад в систему отсчета, связанную с землей, заключаем, что при отсутствии проскальзывания между нижней точкой колеса и дорогой угловая скорость колеса $\omega = v / R$, а все точки поверхности имеют разные скорости относительно земли: например, нижняя точка — нулевую, верхняя $2v$ и т. д.

А пусть водитель при таком движении машины нажимает на педаль газа. Он заставляет колесо вращаться быстрее, чем нужно при данной скорости машины. Колесо стремится проскользнуть назад, возникает сила трения, направленная вперед, которая и разгоняет машину (машина как бы отталкивается от шероховатостей дороги, используя силу трения). Если водитель нажимает на педаль тормоза, колесо стремится вращаться медленнее, чем нужно при данной скорости машины. Возникает сила трения, направленная назад, которая тормозит машину. Если водитель поворачивает колеса машины, возникает сила трения, направленная в сторону поворота, которая машину поворачивает. Таким образом, управление машиной — разгоном, торможением, поворотом — основано на правильном использовании силы трения, причем, конечно, подавляющее большинство водителей об этом даже не догадываются.

Ответим теперь на вопрос: равна ли эта сила своему максимальному значению? Вообще говоря, нет, поскольку нет скольжения колеса относительно дороги, а сила трения равна максимальному значению при скольжении. В покое сила трения может принимать любые значения от нуля до максимального $\mu N$, где $\mu$ — коэффициент трения; $N$ — сила реакции опоры. Поэтому если мы разгоняемся (сила трения направлена вперед), но хотим увеличить темп разгона, мы сильнее нажимаем на педаль газа, и увеличиваем силу трения. Аналогично, если мы тормозим (сила трения направлена назад), но хотим увеличить степень торможения, мы сильнее нажимаем на тормоз и увеличиваем силу трения. Но ясно, что ее можно увеличить и в том и в другом случае, если она не была максимальной! Таким образом, для управления машиной сила трения не должна равняться максимальному значению, и эту разность мы используем для совершения тех или иных маневров. И любой водитель (даже если он ничего не знает про силу трения, а таких, конечно, подавляющее большинство) интуитивно чувствует, есть ли у него резерв силы трения, «далеко» ли машина от пробуксовки, и есть ли возможность ей управлять.

Тем не менее, есть одна ситуация, когда сила трения равна своему максимальному значению. Эта ситуация называется заносом. Пусть водитель резко затормозил на скользкой дороге. Машина начинает скользить по дороге, это состояние движения и называется заносом. В этом случае сила трения направлена противоположно скорости (назад) и равна своему максимальному значению. Это ситуация очень опасна, ведь машина АБСОЛЮТНО неуправляема. Мы не можем повернуть (хоть как-то, хоть чуть-чуть), ведь для поворота нам нужна сила трения, направленная в сторону поворота, а в нашем распоряжении ее нет — сила трения максимальна и направлена назад. Мы не можем увеличить скорость торможения (невозможно увеличить силу трения — она и так максимальна), не можем (даже если бы мы захотели этого в такой ситуации) ускориться. Мы не можем ничего! Ситуация осложняется еще и тем, что в состоянии заноса машину никто не «держит» на дороге. Почему машина в обычных условиях не съезжает в кювет, ведь полотно дороги всегда делается покатым к обочинам, чтобы стекала вода? Ее держит сила трения, а вот если машина скользит (занос) сила трения направлена противоположно скорости и никак иначе. Поэтому любое «боковое» возмущение — покатость дороги, небольшой камень под одним из колес — могут развернуть или сбросить машину на обочину. Никогда не допускайте заноса1.

Теперь сравним мощность, которую могут развивать на дороге передне- и заднеприводной автомобили с одинаковым мотором. Очевидно, что мощность, которую может развивать автомобиль на дороге, зависит не только от его двигателя, но и от того, как автомобиль «использует» силу трения. Действительно, в отсутствие силы трения автомобиль стоял бы на месте (с вращающимися колесами) независимо от мощности двигателя (вращающего эти колеса). Докажем, что заднеприводные автомобили мощнее переднеприводных при одинаковой мощности мотора и оценим отношение мощностей, которые может развивать двигатель, разгоняя машину на дороге (при условии, что мощность самого двигателя может быть очень большой).


Разгоняет автомобиль сила трения, действующая на ведущие колеса, а она не может превышать значения $\mu N$ ($N$ — сила реакции). Поэтому чем больше сила реакции, тем больших значений может достигнуть разгоняющая сила трения (а нажатие на педаль газа в ситуации, когда сила трения достигла максимума, приведет только к проскальзыванию и к заносу, но не к увеличению мощности, которую развивает двигатель). Найдем силы реакции для задних и передних колес машины. Силы, действующие на машину при разгоне, показаны на рисунках (на правом — для заднеприводной, на левом — для переднеприводной). На машину действуют: сила тяжести, силы реакции и сила трения. Поскольку машина движется поступательно, сумма моментов всех сил относительно ее центра тяжести равна нулю. Поэтому, если центр тяжести машины находится точно посередине машины, расстояние между задними и передними колесами $l$, а высота центра тяжести над дорогой $h$, условие равенства нулю суммы моментов относительно центра тяжести дает (при условии, что машина движется, развивая максимальную мощность на максимуме силы трения):

переднеприводная машина

$N_{1} \frac{l}{2} = N_{2} \frac{l}{2} + F_{тр} h = N_{2} \frac{l}{2} + \mu N_{2} h$, (1)

заднеприводная машина

$N_{1} \frac{l}{2} = N_{2} \frac{l}{2} + F_{тр} h = N_{2} \frac{l}{2} + \mu N_{1}h$, (2)

где $\mu$ — коэффициент трения. {(зп)}} = 0,85$.








СОБСТВЕННО, на скользкой дороге вам грозят три основные опасности.

Снос — увод передней оси (или всего автомобиля) с заданной траектории наружу поворота. Машина рискует вылететь с проезжей части.

Занос — увод с траектории задней оси. Может вызвать неконтролируемое вращение автомобиля.

Скольжение — явление, когда все четыре колеса теряют контакт с дорожным покрытием и машина начинает неконтролируемо скользить по прямой.

Но прежде чем переходить к советам по борьбе с заносом и пр., необходимо упомянуть о других факторах, которые не имеют прямого отношения к расположению ведущих колес, однако во многом определяют поведение автомобиля на зимних трассах.

Покрышки

КАКИЕ зимние шины выбрать — с шипами или без? Многие водители, проживающие в больших городах, предпочитают нешипованную резину. Ведь снег с улиц мегаполисов убирают достаточно неплохо, и большую часть времени вы ездите по чистому, разве что влажному асфальту. В таких условиях нешипованная покрышка по меньшей мере комфортнее, в первую очередь — тише. Однако если машина неожиданно попадает на участок, покрытый льдом, то здесь все преимущества — за шипами; они преодолеют подобное препятствие практически без проблем. Зато у шипованных покрышек помимо большей шумности есть и другие минусы. Они, как правило, менее предсказуемы в управлении и резче срываются в юз на мокром асфальте. А восстанавливая сцепление со снегом или льдом, шипованная резина прекращает скольжение также более резко и порой неожиданно для водителя. Все эти нюансы надо учитывать при езде на том или ином типе зимних покрышек.

Антиблокировочная система

СОВРЕМЕННЫМ автолюбителям уже не надо объяснять, что АБС позволяет машине даже при аварийном торможении сохранить управляемость, тем самым давая водителю шанс сманеврировать и объехать препятствие. Если ваша машина не оборудована АБС (что сегодня уже редкость), помните, что тормозить надо аккуратно, не давая колесам заблокироваться. В противном случае автомобиль станет неуправляемым и поедет по прямой, куда бы вы ни поворачивали руль. Почувствовав начало скольжения, немного отпустите педаль, снова прижмите — и дальше тормозите так же прерывисто, на грани блокировки шин.

А вот при наличии АБС действовать следует строго наоборот — давить на тормоз что есть силы, не обращая внимания на треск и вибрацию педали. Только так замедление будет наиболее эффективным и автомобиль остановится максимально быстро.

Электронная система стабилизации

НА МАШИНАХ разных марок она может называться по-разному: ESP, DSC и т.д. Это не суть важно, поскольку назначение у этих систем одно — стабилизировать курсовую устойчивость автомобиля. А важно то, что владельцы автомобилей, оснащенных ESP, зачастую ведут себя на дороге излишне самоуверенно — мол, электроника всегда выручит. Увы, далеко не всегда. Система стабилизации способна исправить небольшие огрехи пилотирования, но если вы совершите серьезную ошибку, электроника может оказаться бессильной… Однако для неискушенного водителя наличие подобных страхующих устройств — большое благо. В ряде случаев они могут хотя бы частично компенсировать недостаток опыта: притормозить буксующее колесо, временно ограничить крутящий момент на ведущих колесах, довернуть руль на небольшой угол.. Причем в идеальном случае человек, сидящий за рулем, даже не успевает осознать, что допустил небольшой огрех в пилотировании, который тут же исправил электронный помощник.

Тип коробки передач

НА ВОПРОС, какой тип коробки передач лучше, однозначного ответа нет. У “механики” и “автомата” есть свои достоинства и недостатки. К примеру, МКПП позволяет при умелом обращении интенсивнее тормозить двигателем, быстрее переключать передачи, вовремя “воткнуть” ту скорость, которая в данный момент более предпочтительна. А в экстренных ситуациях “механика”, допускает быстрое переключение на более низкую передачу для увеличения тяги на ведущих колесах.

Автоматические трансмиссии, несмотря на некоторую задержку при переключениях, несомненно, удобнее для повседневной езды по городу. Тем не менее владельцам автомобилей с АКПП следует учитывать, что в ряде случаев электроника переключает передачи по своему усмотрению. В том числе в поворотах, что из-за кратковременной потери тяги на ведущих колесах может привести к потере устойчивости на скользком покрытии. Чтобы этого не произошло, следует задействовать специальный зимний режим работы трансмиссии (на тех машинах, где он предусмотрен) или ограничить диапазон используемых передач (в зависимости от конструкции коробки).

ПЕРЕДНИЙ ПРИВОД

Борьба с инстинктами

Об особенностях управления переднеприводным автомобилем в зимних условиях рассказывает тренер-инструктор школы искусства вождения “Driving Art” Максим Кузенов.

ПРИ ДВИЖЕНИИ по прямой машина с передними ведущими колесами ведет себя достаточно уверенно даже на скользкой дороге. В результате водитель перестает придавать значение состоянию покрытия, думая, что все под контролем, расслабляется, а когда впереди появляется поворот — входит в него на слишком большой скорости. Переднеприводный автомобиль в этом случае начинает скользить передней осью наружу виража. То есть его сносит.

Начинающий водитель, не готовый к такому развитию событий, обычно пытается “запихнуть” машину в поворот, поворачивая руль на больший угол, чем требуется. Но тем самым он только усугубляет ситуацию. Более опытные автомобилисты часто совершают другую ошибку: слишком резко нажимают педаль газа, стремясь “вытянуть” автомобиль на правильную траекторию. Это тоже больше вредит, чем помогает. Избыток тяги заставляет колеса буксовать, их сцепление с дорогой ухудшается, и снос усиливается.

Как этого избежать? Главное — не паниковать. Почувствовав, что передние колеса начали скользить наружу поворота, не пытайтесь увеличить угол поворота руля. Инстинкт требует именно таких действий, но ничего хорошего из них не выйдет. Действовать надо наоборот — немного выпрямить колеса (тогда их сцепление с дорогой быстрее восстановится), а уже после этого возвращать автомобиль на верную траекторию. Хотя психологически очень сложно (как ни странно, особенно для водителей с большим стажем вождения) повернуть баранку в ту сторону, куда машину и так уже несет…

Вообще помните, что любые резкие движения на скользкой дороге крайне нежелательны. Они нарушают баланс машины. Поэтому руль надо поворачивать быстро, но плавно. Без рывков. В противном случае вы рискуете спровоцировать так называемый “динамический хлыст” — неконтролируемые колебания автомобиля, которые приведут к полной потере контроля над ним.

Одновременно с работой рулем следует немного уменьшить подачу топлива и даже, может быть, слегка нажать на тормоз или выключить и тут же включить сцепление. Тем самым вы дополнительно загрузите передние колеса, и они снова зацепятся за дорогу.

Рассмотрим другой вариант развития событий — когда, испугавшись высокой скорости или заметив какое-либо препятствие, водитель в повороте резко отпускает педаль газа и нажимает на тормоз. В этом случае вместо сноса передней оси начинается занос задней. Сила инерции преобразуется в центробежную силу, стремящуюся развернуть машину вокруг передних колес. Задняя ось начинает скользить наружу поворота. (К таким же последствиям может привести и переключение передач в тот момент, когда автомобиль едет по дуге. )

Чтобы выйти из такой ситуации, в первую очередь надо повернуть руль в сторону заноса. То есть в том направлении, куда вы хотите поехать. При этом надо слегка (ни в коем случае не до пола!) нажать педаль газа, чтобы передние ведущие колеса вытянули машину из заноса. В тот момент, когда автомобиль начнет выравниваться, поставьте руль прямо. Но газ не отпускайте! Таким образом, для успешного выхода из заноса вам снова придется побороть подсознательное стремление отпустить педаль газа и экстренно затормозить.

Есть еще несколько важных психологических моментов. Если, к примеру, машина начала скользить и ее несет в сторону обочины, не смотрите туда! Взгляд водителя всегда должен быть направлен в том направлении, куда он намеревается ехать.

Если чувствуете, что потеряли контроль над ситуацией и справиться с машиной не удается, не продолжайте безуспешные попытки. Неумелые действия обычно приводят к самым серьезным авариям. Лучше поставьте руль в нулевое положение, нажмите на тормоз и ждите. Если повезет, через некоторое время автомобиль сам стабилизируется и остановится…

Всегда помните, что в вираже надо двигаться на постоянном газу. Выбирать правильную передачу и скорость надо еще до начала поворота, пока автомобиль движется по прямой. Рисковать не стоит. Я советую совершать маневр на заведомо заниженной скорости — лучше перестраховаться и пройти поворот не так быстро, как хотелось бы, зато безопасно.

Если автомобиль оборудован “механикой”, не рекомендую постоянно держать левую ногу над педалью сцепления. В критической ситуации вы можете непроизвольно нажать на педаль и из-за этого потерять контроль над машиной.

Наконец, имейте в виду, что даже опытные водители порой совершают ошибки. Поэтому лучше не полагаться на свое умение выходить из критических ситуаций, а вообще не попадать в них.

Почувствовав, что передние колеса начали скользить, не пытайтесь увеличить угол поворота руля!

ЗАДНИЙ ПРИВОД

Главное — не горячиться

Своим опытом управления заднеприводным автомобилем в зимних условиях делится шеф-инструктор школы водительского мастерства BMW Денис Вагин.

НА СНЕГУ, на льду или на чистом асфальте заднеприводный автомобиль ведет себя практически одинаково. Незначительные отличия лишь в интенсивности действий водителя, которые должны соответствовать надежности сцепления колес с дорожным покрытием. Тем не менее зимой на заднеприводном автомобиле ездить сложнее. Хотя бы потому, что задняя часть у большинства современных машин немного легче передней, а соответственно ведущие колеса слабее прижимаются к дороге. Именно поэтому при езде по снегу и льду первые проблемы начинаются еще при трогании с места. Избыток газа и резкий бросок сцепления — ошибка довольно распространенная. Из-за этого в лучшем случае ведущие колеса просто забуксуют. Хуже, если машину развернет поперек дороги. Поэтому резких движений следует избегать уже при старте.

В остальном в черте города, где снег худо-бедно чистят, зимняя езда почти не отличается от летней. Только скорость стоит держать поменьше, чем в теплое время года, а дистанцию с другими машинами — побольше. А вот обледеневшая загородная трасса зачастую становится крайне опасной. Неосторожное прибавление газа в любой момент может привести к заносу. Прохождение знакомого поворота с привычной по лету скоростью зимой чревато сносом передней оси. В обоих случаях автомобиль стремится выйти из-под контроля. Резкое торможение только усугубляет ситуацию. Что же делать?

В любом случае надо помнить, что для заднеприводного автомобиля снос значительно опаснее заноса. Почему? Потому, что восстановить связь с дорогой передних колес, не имеющих тяги, очень сложно. Почти невозможно, пока автомобиль полностью не остановится. Стоит начаться сносу, как машина превращается в неуправляемый снаряд. Чтобы избежать этого, не поворачивайте руль в поворотах на слишком большой угол. Лучше его немного не докрутить, чем перекрутить и потерять контроль над автомобилем.

Если все же снос начался, то в первую очередь следует снять ногу с педали газа. Это просто, поскольку и рефлекс того же требует. При этом не стоит делать резких маневров рулем. Больше вообще ничего предпринимать не надо — только ждать, когда передние колеса вновь зацепятся за дорогу. Рано или поздно (разумеется, счет идет на доли секунд) это произойдет, главное — этому не мешать. И ни в коем случае не пытаться (вот здесь придется преодолеть инстинкт!) довернуть руль на больший угол — снос от этого только усилится.

Иное дело занос. С ним на заднеприводной модели вполне можно бороться активно. В самом начале заноса достаточно лишь немного уменьшить газ и плавно выровнять рулем траекторию автомобиля. В этом случае физика процесса не противоречит рефлекторным действиям водителя. Главное — не суетиться. Своевременный поворот руля зачастую приводит к успеху. Конечно, это требует определенных навыков и изрядной доли хладнокровия. Паника может обойтись дорого.

В частности, важно перебороть себя и воздержаться от удара по тормозам и сцеплению — это только ухудшит связь колес с дорогой, и последствия могут быть плачевными. Повторю: надо просто отпустить педаль газа, чтобы задние колеса подтормаживались двигателем, — это поможет стабилизировать автомобиль. После чего можно попытаться выровнять машину, поворачивая руль (очень плавно!) сначала в сторону заноса, а затем в противоположную, возвращая автомобиль на заданную траекторию. И еще раз: главное — не паниковать и не предпринимать резких торможений!

Передачу ни в коем случае нельзя выключать и при спуске с горы. В этом случае тоже безопаснее всего замедляться с помощью двигателя. Тормозами надо пользоваться очень осторожно, периодически приотпуская педаль, чтобы не допустить блокировки колес. На обледеневшем склоне, едва хотя бы одно колесо заскользит, пиши пропало — машину начнет крутить.

Помните, что для заднеприводного автомобиля снос значительно опаснее заноса!

ПОЛНЫЙ ПРИВОД

Следите за балансом

О нюансах зимней езды на полноприводных автомобилях рассказывает шеф-инструктор школы водительского мастерства “quattro”, заслуженный мастер спорта, обладатель кубка Европы по ралли Евгений Васин:

ЧАСТО приходится слышать, что в критической ситуации полноприводная машина менее понятна в управлении, чем автомобили с приводом на одну ось. Отчасти с этим можно согласиться, однако примите во внимание: при прочих равных условиях эта самая критическая ситуация у моделей с трансмиссией “4х4” возникает на гораздо более высокой скорости, чем у моноприводных машин. Автомобили с двумя ведущими осями имеют момент-тягу (терминология, принятая в школе “quattro”. — Прим. ред.) на всех четырех колесах, что делает их более стабильными как при движении по прямой, так и на дуге поворота.

С другой стороны, в экстремальной обстановке у полноприводной машины могут проявиться особенности поведения, свойственные автомобилям как с задним, так и с передним приводом. Поэтому на моделях с трансмиссией “4х4” очень важно чувствовать распределение момента-тяги между передней и задней осями. Пробуксовка передних колес вызывает скольжение передней части автомобиля наружу поворота — как на переднеприводных моделях. А буксующие задние колеса, наоборот, срывают его в занос — как заднеприводные автомобили.

Чтобы избежать сноса, прежде всего надо еще до начала маневра выбрать оптимальную скорость, позволяющую машине оставаться на заданной траектории. Но если передние колеса все же заскользили наружу поворота, ни в коем случае не оставляйте их без тяги — надо пытаться, насколько возможно, плавно увеличивать газ. Самое главное — в таких обстоятельствах нельзя резко тормозить или сбрасывать газ. Как только на колесах пропадает тяга, автомобиль едет туда, куда его тащит сила инерции. Если все же пришлось тормозить, то делать это надо очень аккуратно, дозируя усилие на педали тормоза, желательно не доводя до срабатывания АБС.

Самая распространенная ошибка при сносе — поворот руля на больший угол, чем требуется. Наши инструкторы называют такие действия “поставить колеса плугом”. При этом передняя ось смещается наружу и машина выезжает на соседнюю полосу или, хуже того, на столб, дерево, встречное транспортное средство.. На скользком покрытии ни в коем случае нельзя поворачивать баранку на большие углы. Понимание этого приходит лишь с опытом. Между прочим, поэтому в нашей школе нет статических тренажеров для обучения скоростному рулению — все навыки ученики отрабатывают на практике и только в динамике.

Занос полноприводной машины, пожалуй, возникает чаще, чем снос. Особенно зимой на скользком покрытии. Рекомендации по предотвращению заноса общеизвестны: уменьшить скорость перед вхождением в поворот, плавно работать тормозом и газом, аккуратно и на небольшие углы вращать руль. Ну а если вы все же что-то не учли или не так рассчитали, и автомобиль начало разворачивать? Тогда необходимо одновременно с плавным добавлением тяги повернуть руль в сторону заноса и сделать это как можно быстрее.

Вообще, рекомендации, как вести себя на зимней дороге, сводятся к тем же простым правилам: перед любым участком дороги, который вы расцениваете как потенциально опасный, надо заранее принять упреждающие меры. Сбрасываете скорость, выставляете автомобиль на заданную траекторию движения, аккуратно действуете рулем и газом. Вот, собственно говоря, и все.

Правда, жизнь полна неожиданностей, и, несмотря на всю вашу осторожность, машина все же может попасть в критическую ситуацию, исправить которую водителю не под силу. Что делать в таком случае? Мнения специалистов расходятся. Одни считают, что лучше ничего не предпринимать и положиться на волю судьбы, дабы не усугублять и без того опасное положение. Другие предпочитают бороться до конца, используя все свои силы и навыки. Третьи советуют попытаться погасить скорость: тормозить, дотормаживать, включать низшую передачу.. Конечно, универсальных советов нет и быть не может. Все зависит от конкретной ситуации. Но лично я всегда стараюсь придерживаться тактики активного участия в управлении автомобилем, полагаясь на свои знания и опыт.

На полноприводных моделях главное — правильно чувствовать распределение тяги между осями.

Автор Издание Клаксон №24 2007 год Фото Фото “Audi”, BMW и Алексея БАРАШКОВА

СИЛА ТРЕНИЯ И СИЛА СОПРОТИВЛЕНИЯ ДВИЖЕНИЮ

Задание 400.

Зачем в гололедицу тротуары посыпают песком?

Ответ

Задание 401.

Зачем зимой задние колеса некоторых грузовых автомобилей перевязывают цепями?

Ответ

Задание 402.

Для чего при спуске воза с горы одно колесо телеги иногда закрепляют так, чтобы оно не вращалось?

Ответ

Задание 403.

Зачем на шинах автомашин, колесных тракторов делают глубокий рельефный рисунок (протектор)?

Ответ

Задание 404.

Зачем осенью у трамвайных линий, проходящих около парков, бульваров и садов, вывешивается предупреждающий знак «Осторожно, листопад!»?

Ответ

Задание 405.

Почему после дождя грунтовая дорога скользкая?

Ответ

Задание 406.

Почему после дождя опасно съезжать на автомобиле по грунтовой дороге под уклон?

Ответ

Задание 407.

Зачем некоторые мастера смазывают мылом шуруп перед ввинчиванием его в скрепляемые детали?

Ответ

Задание 408.

Зачем стапеля, по которым судно спускают в воду, обильно смазывают?

Ответ

Задание 409.

Для чего делается насечка около шляпки гвоздя?

Ответ

Задание 410.

Назовите одну-две детали велосипеда, изготовленные с учетом увеличения силы трения скольжения.

Ответ

Задание 411.

Какие силы трения возникают при движении карандаша в случаях, указанных на рисунке 93, а, б? Куда направлена сила трения, действующая на карандаш, относительно оси карандаша в обоих случаях?

Рис.93

Ответ

Задание 412.

Тележка с грузом движется (рис. 94). Какой вид трения возникает между:

А) столом и колесами;
б) грузом и тележкой;
в) осями колес и корпусом тележки?

Рис.94

Ответ

Задание 413.

Почему кирпичи не скатываются вниз (рис. 95 и 96)? Какая сила удерживает их в состоянии покоя? Изобразите силы, действующие на кирпичи.

Рис.95

Рис.96

Ответ

Задание 414.

Брусок двигают вправо (рис. 97). Куда направлена сила трения скольжения по отношению к бруску; относительно поверхности, по которой движется брусок?

Рис.97

Ответ

Задание 415.

Лестница у стены занимает положение, изображенное на рисунке 98. Укажите направление силы трения в местах соприкосновения лестницы со стеной и полом.

Рис.98

Ответ

Задание 416.

Брусок движется равномерно (рис. 99). Куда направлена:

А) сила упругости горизонтальной части нити;
б) вертикальной;
в) сила трения скольжения, действующая на поверхность стола, на брусок;
г) чему равна равнодействующая этих сил?

Рис.99

Ответ

Задание 417.

Колесо автомобиля буксует (рис. 100). Куда направлена сила трения скольжения между буксующим колесом и дорогой, которая действует:

А) на колесо;
б) дорогу? Куда направлена сила упругости дороги?

Рис. 100

Ответ

Задание 418.

Книга прижата к вертикальной поверхности (рис. 101). Изобразите графически направления сил тяжести и трения покоя, действующих на книгу.

Рис. 101

Ответ

Задание 419.

Тележка равномерно движется вправо (см. ). Какая сила приводит в движение груз, поставленный на нее? Чему равна эта сила при равномерном движении?

Ответ

Задание 420.

На транспортере равномерно движется ящик с грузом (без скольжения). Куда направлена сила трения покоя между лентой транспортера и ящиком, когда ящик:

а) поднимается;
б) движется горизонтально;
в) опускается?

Ответ

Задание 421.

Если автобус равномерно движется по горизонтальному участку пути, чему равна сила трения покоя?

Ответ

Задание 422.

Парашютист, масса которого 70 кг, равномерно опускается. Чему равна сила сопротивления воздуха, действующая на парашютиста?

Ответ

Задание 423.

С помощью динамометра равномерно перемещают брусок (см. ). Чему равна сила трения скольжения между бруском и поверхностью стола? (Цена деления динамометра 1 Н.)

Ответ

Задание 424.

Зубья пилы разводят в разные стороны от плоскости пилы. На рисунке 102 показаны пропилы, сделанные неразведенной и разведенной пилами, Какой пилой труднее пилить: разведенной или неразведенной? Почему?

Рис. 102

Ответ

Задание 425.

Приведите примеры, когда трение приносит пользу и когда вред.

Ответ

Задание 426.

На уроке физкультуры мальчик равномерно скользит вниз по канату. Под действием каких сил осуществляется это движение?

Ответ

Задание 427.

Судно буксирует три баржи, соединенные последовательно одна за другой. Сила сопротивления воды для первой баржи 9000 Н, для второй 7000 Н, для третьей 6000 Н. Сопротивление воды для самого судна 11 кН. Определите силу тяги, развиваемую судном при буксировке этих барж, считая, что баржи движутся равномерно.

Решение и ответ

Задание 428.

На движущийся автомобиль в горизонтальном направлении действуют сила тяги двигателя 1,25 кН, сила трения 600 Н и сила сопротивления воздуха 450 Н. Чему равна равнодействующая этих сил?

Решение и ответ

Задание 429.

Можно ли однозначно утверждать, что приращение силы сопротивления ▲F равно 3 мН, если скорость тела, движущегося в некоторой среде с коэффициентом сопротивления 0,01, увеличилась на 0,3 м/с?

Ответ

Задание 430.

Троллейбус трогается с места и в течение 30 с приобретает импульс 15 10 4 кг м/с. Определите силу сопротивления движению, если развиваемая троллейбусом сила тяги равна 15 кН.

Решение и ответ

Задание 431.

На автомобиль массой 103 кг во время движения действует сила сопротивления, равная 10% от его веса. Чему должна быть равна сила тяги, развиваемая автомобилем, чтобы он двигался с постоянным ускорением 2 м/с 2 ?

Решение и ответ

Задание 432.

Конькобежец вначале движется по горизонтальному пути равномерно, а затем после разгона путь 60 м до остановки проезжает за 25 с. Чему равен коэффициент трения скольжения коньков по льду?

Решение и ответ

Задание 433.

Поезд массой 400 т движется со скоростью 40 км/ч и после торможения останавливается. Какова сила торможения, если тормозной путь поезда равен 200 м?

Решение и ответ

Задание 434.

Велосипедист, ехавший со скоростью 11 м/с, резко затормозил. Коэффициент трения скольжения шин о сухой асфальт равен 0,7. Определите ускорение велосипедиста при торможении; время торможения; тормозной пусть велосипедиста.

Решение и ответ

Задание 435.

Какую силу надо приложить в горизонтальном направлении к вагону массой 16 т, чтобы уменьшить его скорость на 0,6 м/с за 10 с; за 1 с? Коэффициент трения равен 0,05.

Решение и ответ

Задание 436.

С какой скоростью сможет ехать по горизонтальной плоскости мотоциклист, описывая дугу радиусом 83 м, если коэффициент трения резины о почву равен 0,4?

Всем механикам с юности памятна картинка со схемой движения автомобиля по кривой, когда его внешние колеса проходят больший путь, чем внутренние. С ее помощью во многих учебниках для водителей разъясняются назначение и принцип действия дифференциала. Часто все сводится к тому, что дифференциал позволяет ведущим колесам вращаться с различными скоростями и, таким образом, обеспечивает нормальное движение автомобиля на поворотах.

Такие разъяснения не то чтобы совсем неправильны, но слишком упрощены и сути работы дифференциала не раскрывают. Конечно, в серьезных книгах все изложено правильно. Там сказано, что назначение межколесного дифференциала на автомобиле состоит в распределении крутящего момента строго поровну между ведущими колесами одного моста, а межмостового дифференциала – в распределении крутящего момента между ведущими мостами, — поровну или в оптимальной пропорции (несимметричный дифференциал).

«Дифференциал – это механизм, у которого ведущие колеса вращаются независимо друг от дружки».

Строго говоря, вращаются они «зависимо», ну да ладно, — что-то похожее на правду сказано, а об остальном ни слова, чтобы не забивать голову людям без специальной подготовки.

Зеленин С.Ф., Молоков В.А. Учебник по устройству автомобиля, М., «Русьавтокнига», 2000 г., 80 с. Тираж 15000 экз.

Цитата из этой книги:

« Дифференциал предназначен для распределения крутящего момента между полуосями ведущих колес при повороте автомобиля и при движении по неровностям дороги. Дифференциал позволяет колесам вращаться с разной угловой скоростью и проходить неодинаковый путь без проскальзывания относительно покрытия дороги.

Иными словами 100% крутящего момента, который приходит на дифференциал, могут распределяться между ведущими колесами как 50 х 50, так и в другой пропорции (например, 60 х 40). К сожалению, пропорция может быть и 100 х 0. Это означает, что одно из колес стоит на месте (в яме), а другое в это время буксует (по сырой земле, глине, снегу).

Что поделаешь! Ничто не бывает абсолютно правильным и идеальным, зато данная конструкция позволяет автомобилю поворачивать без заноса, а водителю не менять каждый день напрочь изношенные шины.

Рис. 38 Главная передача с дифференциалом

1 — полуоси; 2 — ведомая шестерня; 3 — ведущая шестерня; 4 — шестерни полуосей; 5 — шестерни-сателиты

Это уже не упрощение, а просто введение в заблуждение читателей. Здесь, кроме второго предложения и иллюстрации, все неправда (в первом предложении нужно вставить слово «поровну», а точку поставить после слова «колес» и т.д.).

Только однажды в учебнике для профтехобразования мне довелось встретить правильное и при этом простое и наглядное разъяснение сути работы дифференциала. Было это давно и помню только, что это был учебник для водителей зерновых комбайнов.

Там читателю предлагалось вообразить, что две полуосевые конические шестерни «развернуты» в две зубчатые рейки, эти рейки лежат на воображаемом столе, а между ними помещен сателлит в виде прямозубой шестерни. Выглядит это примерно так:

Объяснение сути работы дифференциала основано на его конструкции и на третьем законе Ньютона, который гласит: сила действия равна по модулю и противоположна по направлению силе противодействия. На следующем рисунке показано силовое взаимодействие сателлита с рейками, когда движущая сила Д приложена к оси сателлита и этот сателлит толкает обе рейки по столу, причем силы сопротивления движению левой и правой реек С лев и С прав одинаковы (силы трения реек о поверхность воображаемого стола) и каждая из них равна половине общей силы сопротивления С. Силы со стороны сателлита передаются на рейки в точках зацепления зубьев сателлита с зубьями реек. Благодаря равенству сил сопротивления движению С лев и С прав, равны между собой и движущие силы на зубьях сателлита, каждая из которых равна половине движущей силы Д. Поскольку равные силы приложены к двум зубьям сателлита, находящимся на равных расстояниях от его оси, сателлит находится в равновесии и не вращается. Поэтому все три детали движутся прямолинейно в одну сторону и с равными скоростями, а именно с той скоростью, с какой движется ось сателлита и которая задана двигателем.

Эта ситуация соответствует установившемуся движению автомобиля по дороге с хорошим сцеплением с дорогой.

Теперь представим, что при своем движении по столу, левая рейка «наехала» на пятно масла. При этом сила сопротивления ее движению (сила трения о стол) уменьшилась, а сила сопротивления движению правой рейки осталась прежней. На какой-то момент равновесие сил на зубьях сателлита нарушается: нагрузка на левый его зуб становится меньше нагрузки, действующей на его правый зуб. Иначе говоря, сателлиту стало легче толкать левую рейку, чем правую. Поэтому он начинает вращаться по часовой стрелке, как это показано на следующем рисунке.

Благодаря вращению сателлита движение правой рейки замедляется, а левая рейка наоборот ускоряется. Затем правая рейка полностью останавливается, а сателлит продолжает вращаться. Его ось продолжает двигаться с той же скоростью, что и прежде, так как эта скорость задана двигателем. Но поскольку правая рейка стоит, вращающийся сателлит обкатывается по ней. В момент, показанный на рисунке правый зуб сателлита стоит на месте, так как «упирается» в зуб неподвижной рейки. Но противоположный, левый зуб сателлита движется в два раза быстрее, чем ось самого сателлита. Все это соответствует ситуации, когда одно из ведущих колес медленно движущегося автомобиля наезжает, например, на обширное пятно льда, а второе остается на сухом покрытии с хорошим сцеплением. То есть машина останавливается и колесо, находящееся на льду, буксует, вращаясь в два раза быстрее, чем прежде, когда оба колеса катились с одинаковой скоростью.

Строго говоря, о нарушении равновесия сил на зубьях сателлита выше сказано некорректно и только потому, что, как мне кажется, так проще понять происходящее. На самом деле равновесие сил сохраняется всегда, только для его рассмотрения нужно еще учитывать силы, вызывающие ускорение левой рейки и замедление правой. Эти не рассматриваемые нами силы, исчезают с момента полной остановки правой рейки. В этот же момент удвоенная скорость движения левой рейки становится постоянной. И тогда ситуация полностью соответствует следующему рисунку.

Здесь равновесие сил восстановилось, точнее, — исчезли динамические силовые составляющие (те, что вызывали ускорение одной рейки и замедление другой). Правая рейка стоит, сателлит вращается, а левая рейка движется равномерно с удвоенной скоростью. Очень важно отметить что, равновесие сил перешло на новый уровень. Теперь равные силы на левом и правом зубьях сателлита стали существенно меньше прежних. В силу третьего закона Ньютона эти силы не могут превысить движущую силу, которую можно приложить к рейке, находящейся на пятне масла, или к колесу, находящемуся на пятне льда. Иными словами, если одно колесо стоит на сухой дороге, а противоположное буксует на льду или в грязи, это вовсе не означает, что 100% крутящего момента передается от двигателя на буксующее колесо, как сказано в упомянутой выше книге. Этот момент всегда и во всех условиях делится дифференциалом поровну между колесами, но он не может быть больше, чем позволяет сцепление одного из колес с дорогой, причем именно того колеса, у которого это сцепление меньше.

Только если в этих условиях заблокировать дифференциал, то есть выключить его из работы, тем или иным способом жестко соединив между собой полуоси, можно передать на колесо, стоящее на сухой дороге, подавляющую часть крутящего момента, который может развить двигатель. При этом буксование прекратится, оба колеса будут вращаться с одинаковой скоростью, но подавляющую часть суммарной силы тяги будет обеспечивать только одно из этих колес.

Мне кажется, что с помощью модели с зубчатыми рейками можно наглядно объяснить и все прочие режимы работы межколесного дифференциала. Например, ситуацию, иногда возникающую при торможении двигателем. Представим, что автомобиль движется под уклон на сухой дороге с пятнами льда. Водитель тормозит двигателем. В этом случае движущая сила, это сила инерции массы машины. А сила сопротивления движению, это сила, приложенная к осям сателлитов дифференциала со стороны двигателя. Одно из колес наезжает на пятно льда. Сила сцепления этого колеса с дорогой резко уменьшается, и оно начинает вращаться в обратную сторону. Здесь происходит то же самое, что произойдет с рейками если ось сателлита сделать неподвижной, но оставить ему свободу вращения вокруг этой оси, то есть имитировать ситуацию, когда ось сателлита тормозится или удерживается двигателем. Если теперь двинуть вперед одну из зубчатых реек, то сателлит начнет вращаться и заставит вторую рейку двигаться назад. Здесь рейка, движимая вперед, соответствует колесу на сухой дороге, а рейка, движущаяся назад, — колесу, находящемуся на льду и вращающемуся в обратную сторону. На мой взгляд, вращение буксующего колеса в обратную сторону очень наглядно демонстрирует «стремление» дифференциала выполнить свое предназначение и выровнять силы на двух колесах ведущего моста. В данном случае это силы торможения. Благодаря их выравниваю исключается или сильно снижается вероятность заноса автомобиля при таком режиме торможения.

Можно рассматривать еще многие ситуации, возникающие при работе дифференциала. Но полагаю, что и сказанного достаточно, чтобы убедиться: — межколесный дифференциал всегда делит получаемый от двигателя крутящий момент поровну между двумя колесами одного ведущего моста.

А теперь вернемся к упомянутой в самом начале картинке с автомобилем, движущемся по кривой. Если автомобиль заднеприводной, то получающие одинаковый крутящий момент два задних колеса преобразуют эти крутящие моменты в две одинаковые силы тяги (если шины колес имеют одинаковый диаметр, одинаковое давление накачки и несут одинаковые части веса автомобиля). А две одинаковые силы тяги стремятся толкать автомобиль по прямой. Именно поэтому, водителю при прохождении поворота приходится твердо удерживать рулевое колесо. Строго говоря, дифференциал на таком автомобиле не столько помогает, сколько мешает прохождению поворота. Зато он прямо способствует устойчивости движения по прямой (вместе с углами установки передних колес).

У переднеприводного автомобиля ситуация несколько иная. Здесь силы тяги также одинаковы на двух колесах, но они «поворачиваются» вместе с поворачиваемыми колесами. Поэтому, например, переднеприводной машине легче выйти из глубокой скользкой колеи: повернутые передние ведущие колеса активно тянут куда нужно. А у заднеприводного, задние ведущие колеса активно толкают машину вдоль колеи.

Здесь рассмотрена лишь малая часть того, что следовало бы водителям знать о работе дифференциала и на это потребовалось много слов и картинок. Так может быть правы те, кто ограничивается пресловутой картинкой с разным пробегом у разных колес на повороте? Может быть. Но полагаю, что следует, если и не вдаваться в пространные разъяснения, то хотя бы просто написать, для чего действительно предназначен этот механизм. А кто захочет дойти до сути, найдет, где об этом почитать. И уж совсем ни к чему пропагандировать собственное неверное понимание этой сути.

400. Зачем в гололедицу тротуары посыпают песком?
Для того, чтобы увеличить коэффициент трения. В этом случае вероятность поскользнуться и упасть будет меньше.

401. Зачем зимой задние колеса некоторых грузовых автомобилей перевязывают цепями?
Для того, чтобы увеличить коэффициент трения и тем самым практически не допустить проскальзывания между колесами автомобиля и обледенелым участком полотна дороги.

402. Для чего при спуске воза с горы одно колесо телеги иногда закрепляют так, чтобы оно не вращалось?
Чтобы увеличить трение между телегой и дорогой. В этом случае скорость телеги будет не очень, большой, зато безопасной для спуска.

403. Зачем на шинах автомашин, колесных тракторов делают глубокий рельефный рисунок (протектор)?
Для увеличения коэффициента трения между колесами и дорогой. В этом случае сцепление с землей будет более эффективным.

404. Зачем осенью у трамвайных линий, проходящих около парков, бульваров и садов, вывешивается предупреждающий знак «Осторожно, листопад!»?
Сухие листья уменьшают сцепление колес трамвая с рельсами, вследствие чего может возникнуть пробуксовка колес, тормозной путь трамвая также увеличится.

405. Почему после дождя грунтовая дорога скользкая?
Вода на поверхности земли является смазкой и поэтому уменьшает коэффициент трения.

406. Почему после дождя опасно съезжать на автомобиле по грунтовой дороге под уклон?
Потому что вода на поверхности дороги уменьшает коэффициент трения.

407. Зачем некоторые мастера смазывают мылом шуруп перед ввинчиванием его в скрепляемые детали?
Мыло служит смазкой и уменьшает коэффициент трения. В этом случае процесс ввинчивания шурупа будет более легким.

408. Зачем стапеля, по которым судно спускают в воду, обильно смазывают?
Для того, чтобы уменьшить коэффициент трения между спускаемым судном и стапелями, и тем самым облегчить процесс спуска.

409. Для чего делается насечка около шляпки гвоздя?
Для увеличения коэффициента трения. В этом случае молоток будет меньше соскальзывать со шляпки гвоздя.

410. Назовите одну-две детали велосипеда, изготовленные с учетом увеличения силы трения скольжения.
Резиновая покрышка, тормозные колодки.

411. Какие силы трения возникают при движении карандаша в случаях, указанных на рисунке 93, а, б? Куда направлена сила трения, действующая на карандаш, относительно оси карандаша в обоих случаях?
а) сила трения скольжения; она направлена вдоль оси карандаша в противоположную сторону его движения,
б) сила трения качения; она направлена перпендикулярно к оси карандаша в противоположную сторону его движения.

412. Тележка с грузом движется (рис. 94). Какой вид трения возникает между: а) столом и колесами; б) грузом и тележкой; в) осями колес и корпусом тележки?
а) сила трения качения;
б) сила трения покоя, если груз покоится относительно тележки, или сила трения скольжения, если груз движется;
в) сила трения скольжения.

413. Почему кирпичи не соскальзывают вниз (рис. 95 и 96)? Какая сила удерживает их в состоянии покоя? Изобразите силы, действующие на кирпичи. 

414. Брусок двигают вправо (рис. 97). Куда направлена сила трения скольжения по отношению к бруску; относительно поверхности, по которой движется брусок?
Относительно бруска сила трения скольжения направлена влево (против направления движения). Относительно поверхности, по которой движется брусок, сила трения направлена вправо (по направлению движения).

415. Лестница у стены занимает положение, изображенное на рисунке 98. Укажите направление силы трения в местах соприкосновения лестницы со стеной и полом.

416. Брусок движется равномерно (рис. 99). Куда направлена: а) сила упругости горизонтальной части нити; б) вертикальной части нити; в) сила трения скольжения, действующая на поверхность стола, на брусок? Чему равна равнодействующая этих сил?

417. Колесо автомобиля буксует (рис. 100). Куда направлена сила трения скольжения между буксующим колесом и дорогой, которая действует: а) на колесо; б) на дорогу? Куда направлена сила упругости дороги?

418. Книга прижата к вертикальной поверхности (рис. 101). Изобразите графически направления сил тяжести и трения покоя, действующих на книгу.

419. Тележка равномерно движется вправо (см. рис. 94). Какая сила приводит в движение груз, поставленный на нее? Чему равна эта сила при равномерном движении?
Груз, лежащий на тележке, приводится в движение силой трения покоя, направленной вправо. При равномерном движении те¬лежки эта сила равна нулю.

420. На транспортере равномерно движется ящик с грузом (без скольжения). Куда направлена сила трения покоя между лентой транспортера и ящиком, когда ящик: а) поднимается; б) движется горизонтально; в) опускается?
а) вверх вдоль транспортера; б) она равна нулю; в) вверх вдоль транспортера.

421. Равна ли сила тяги силе трения, если автобус движется без скольжения равномерно: 1) по горизонтальному пути; 2) вверх по наклонному участку пути?
Если автобус движется равномерно по горизонтальному участку пути, то сила трения покоя равна силе тяги за вычетом силы сопротивления воздуха.

422. Парашютист, масса которого 70 кг, равномерно опускается. Чему равна сила сопротивления воздуха, действующая на парашютиста?

423. С помощью динамометра равномерно перемещают брусок (см. рис. 97). Чему равна сила трения скольжения между бруском и поверхностью стола? (Цена деления динамометра 1 Н.)
При равномерном движении бруска сила трения скольжения между бруском и поверхностью стола равна силе упругости пружины динамометра. Поэтому в этом случае динамометр нам показывает значение силы трения скольжения. Согласно рис. 97 она равна 4Н.

424. Зубья пилы разводят в разные стороны от плоскости пилы. На рисунке 102 показаны пропилы, сделанные неразведенной и разведенной пилами. Какой пилой труднее пилить? Почему?
Труднее пилить неразведенной пилой, так как в этом случае боковые поверхности пилы плотнее соприкасаются с деревом и между ними возникает большая сила трения.

425. Приведите примеры, когда трение приносит пользу и когда вред.
Трение приносит пользу при ходьбе, беге, езде на транспорте, движении грузов на транспортере. Трение приносит вред в трущихся деталях различных механизмов, где нежелательно стирание поверхностей.

426. На уроке физкультуры мальчик равномерно скользит вниз по канату. Под действием каких сил осуществляется это движение?
Под действием силы тяжести и силы трения скольжения.

427. Судно буксирует три баржи, соединенные последовательно одна за другой. Сила сопротивления воды для первой баржи 9000 Н, для второй 7000 Н, для третьей 6000 Н. Сопротивление воды для самого судна 11 кН. Определите силу тяги, развиваемую судном при буксировке этих барж, считая, что баржи движутся равномерно.

428. На движущийся автомобиль в горизонтальном направлении действуют сила тяги двигателя 1,25 кН, сила трения 600 Н и сила сопротивления воздуха 450 Н. Чему равна равнодействующая этих сил?

429. Можно ли однозначно утверждать, что приращение силы сопротивления AF равно 3 мН, если скорость тела, движущегося в некоторой среде с коэффициентом сопротивления 0,01, увеличилась на 0,3 м/с?
Однозначно утверждать так нельзя, так как сила сопротивления в вязких средах задается неоднозначно. При малых скоростях движения она пропорциональна скорости, при больших — квадрату скорости.

430. Троллейбус трогается с места и в течение 30 с приобретает импульс 15 104 кг-м/с. Определите силу сопротивления движению, если развиваемая троллейбусом сила тяги равна 15 кН.

431. На автомобиль массой 103 кг во время движения действует сила сопротивления, равная 10% от его веса. Чему должна быть равна сила тяги, развиваемая автомобилем, чтобы он двигался с постоянным ускорением 2 м/с2?

434. Велосипедист, ехавший со скоростью 11 м/с, резко затормозил. Коэффициент трения скольжения шин о сухой асфальт равен 0,7. Определите ускорение велосипедиста при торможении; время торможения; тормозной путь велосипедиста.

435. Какую силу надо приложить в горизонтальном направлении к вагону массой 16 т, чтобы уменьшить его скорость на 0,6 м/с за 10 с; за 1 с? Коэффициент трения равен 0,05.

436. С какой скоростью сможет ехать по горизонтальной плоскости мотоциклист, описывая дугу радиусом 83 м, если коэффициент трения резины о почву равен 0,4?

Силы и реакции, действующие на автомобиль при его торможении

Крутящий момент от дифференциала к ведущим колесам передается валами, называемыми полуосями. Помимо крутящего момента полуоси могут быть нагружены изгибающими моментами. На ведущее колесо автомобиля действуют реакция дороги К, от веса, приходящегося на колесо сила тяги А (при торможении тормозная сила) боковая сила Ку, возникающая при повороте и заносе и т. п. Все эти силы могут создавать изгибающие моменты, которые передаются на полуось. В зависимости от характера установки полуосей в картере моста они могут быть полностью или частично разгружены от изгибающих моментов, возникающих под действием перечисленных сил.  [c.164]
Силы и реакции, действующие на автомобиль при его торможении  [c.415]

По условию Ок = 0. Работа сил, действующих на автомобиль, слагается из суммы работ —силы веса, нормальной реакции и силы торможения  [c.273]

Решение. Автомобиль рассматриваем как материальную точку, на которую действуют (рис. 222) следующие силы (7—сила тяжести автомобиля, К—нормальная реакция дороги и Р — сопротивление от торможения.  [c.305]

Если отключить двигатель от ведущих колес, то автомобиль будет продолжать движение по инерции (накатом). Под действием сил сопротивления движению скорость автомобиля снижается и, наконец, он останавливается. Однако торможение при этом происходит на большом участке пути. Более эффективным является торможение под действием специально создаваемой внешней силы, называемой тормозной. Тормозная сила возникает между колесом и дорогой в результате того, что тормозной механизм препятствует его вращению. Направление тормозной силы противоположно направлению движения автомобиля. Тормозная сила тем больше, чем сильнее тормозной механизм препятствует вращению колеса. Ее максимальное значение зависит от сцепления колеса с дорогой и вертикальной реакции Z, действующей от дороги на колесо  [c.248]

На рис. 195 показана схема сил, действующих на колесо при торможении. На тормозном барабане возникает момент трения или тормозной момент, противодействующий вращению колеса, а между колесом и дорогой появляется реакция, противодействующая движению автомобиля. Эта реакция и называется тормозной силой.  [c.296]

Наиболее вероятным и опасным (с учетом вероятности торможения) является занос задней оси. Из рассмотрения схемы сил, действующих на автомобиль при повороте, можно установить, что занос передней оси исключен, поскольку в этом случае увеличивается радиус поворота, что приводит к уменьшению поперечной силы Ру и касательной реакции Рь При снижении силы Р1 до значения, определяемого условием (82), занос прекращается. При заносе задней оси радиус поворота, наоборот, уменьшается, сила Ру резко возрастает, что вызывает еще больший поворот автомобиля, так как сила Р2, согласно уравнению (79), растет и ее значение больше предельно реализуемого.  [c.233]

По условию tv = 0. Работа сил, действующих на автомобиль, слагается из работ силы тялнормальной реакции и силы торможения  [c.215]

Необходимость изменения соотношения тормозных сил определяется изменением нормальных (вертикальных) реакций на колесах отдельных осей, так как по мере увеличения интенсивности торможения за счет действия горизонтальных сил инерции эти реакции возрастают на передней оси (2,) и уменьшаются на задней (/г). Так, для двухосных автомобилей (рис. 1.1, а)  [c.22]


При торможении автомобиля сила инерции Р , действуя на плече Лц (см. рис. 72), вызывает перераспределение нормальных нагрузок между передними и задними колесами нагрузка на передние колеса увеличивается, а на задние, наоборот, уменьшается. Поэтому значения нормальных реакций Zl и действующих соответственно на передние и задние колеса автомобиля во время торможения, значительно отличаются от значений нагрузок которые они  [c.169]

Так как распределение общей тормозной силы между Колесами не соответствует изменяющимся во время торможения нормальным реакциям на них, то действительный минимальный тормозной путь оказывается на 20—40% больше теоретического. С целью приближения результатов расчета к экспериментальным данным в формулы вводят коэффициент Кд, который учитывает степень использования полной теоретически возможной эффективности действия тормозной системы. Величина коэффициента эффективности торможения в среднем равна 1,2 для легковых автомобилей и  [c.170]

Схема сил, действующих на поворотный кулак и балку управляемого неведущего моста в случае торможения автомобиля, представлена на рис. ХП.4. Высокие нагрузки на балку переднего управляемого моста и поворотный кулак будут иметь место при торможении и боковом заносе автомобиля. В первом случае на колесо действуют вертикальная реакция со стороны дороги и касательная тормозная сила = 2 9, ах. направленная против движения.  [c.297]

Одновременно с осознанием опасности возбуждается эмоциональная сфера человека водитель может испытывать замешательство, растерянность, испуг, удивление. Но осознание опасности и психическая реакция на нее вызывают, как правило, следующее действие водитель нажимает на тормозную педаль совсем не так, как делает это при служебном торможении, а изо всей силы. Так как тормозные механизмы у современных автомобилей весьма мощные, то не только на мокрых и скользких, но и на сухих дорогах колеса блокируются и автомобиль, обладающий значительным запасом кинетической энергии, под влиянием сил инерции продолжает скольжение юзом. Когда препятствие неумолимо приближается к заторможенному таким образом автомобилю, водителю хочется еще сильнее нажать на педаль, хотя она и так уже выжата до отказа. Дальнейшее нажатие на педаль при заблокированных колесах не повышает эффективности торможения, а только еще более закрепляет блокировку колес.  [c.188]

Во время выполнения динамических измерений при испытаниях автомобилей было получено максимальное ускорение в вертикальном направлении, равное 3g. В горизонтальной плоскости поперечные силы, возникающие при движении на повороте, и продольные тормозные силы ограничены сцеплением шины с дорогой, поэтому предельное значение замедления, равное Ig, приемлемо. Гарретт предложил, для нахождения соответствующих максимальных нагрузок умножать величину ускорения (или замедления) на коэффициент запаса, равный 1,5. Таким образом, максимальные вертикальные (удар о препятствие) ускорения составляют 4,5g, продольные (торможение и ускорение) — 1,5 , ускорения при движении на левом или правом повороте достигают l,5g. Случай удара о препятствие рассмотрен на рис. 1.10. Когда автомобиль расторможен, направление равнодействующей силы может проходить только через ось вращения колеса. Если вертикальная статическая реакция, действующая на колесо, равна R, то динамическая реакция будет равна 4,5/ . Равнодействующая сила пройдет через точку контакта колеса с препятствием и через ось колеса и составит Р = = 4,5/ / os0. Горизонтальная составляющая равнодействующей силы будет равна произведению 4,5/ sin0/ os 0 = 4,5/ tg 0. Если препятствие преодолевается так быстро, что кузов автомобиля успевает лишь незначительно приподняться, то эффективная высота препятствия будет равна разности Н — (D—S), где S — статический прогиб (под действием веса автомобиля) подвески D — полная деформация подвески. Высоту препятствия Н обычно принимают равной 150 мм (допустимая деформация шины).  [c.28]

Крутящий момент от диф4)еренциала к ведущим колесам передается валами, называемыми полуосями. Помимо крутящего момента, полуоси могут быть нагружены изгибающими моментами. На ведущее колесо автомобиля действуют реакция дороги R от веса G, приходящегося на колесо (рис. 118, а) сила тяги Р,. (при торможении тормозная сила) боковая сила S, возникающая при повороте и заносе и т. п. Все эти силы могут создавать изгибающие моменты, которые передаются на полуось. В зависимости от характера установки полуосей в картере моста они могут быть полностью или частично разгружены от изгибающих моментов, возникающих под действием перечисленных сил. Если полуось непосредственно опирается на подшипник, установленный в балке заднего моста, то она воспринимает изгибающие моменты от всех перечисленных сил и, кроме того, передает крутящий момент на ведущее колесо. Полуоси такого типа называются полуразгруженными. Полуразгруженные полуоси применены в задних мостах всех легковых автомобилей и грузовых автомобилей малой грузоподъемности (УАЗ-452 и др.).  [c.182]


Для получения максимального значения Рmax тор следует ДСЛЯТЬ все колеса тормозящимг ся, т. е. использовать вертикальные реакции, действующие на все колеса автомобиля. Вертикальные реакции на передних и задних колесах автомобиля меняются вследствие изменения нагрузки, особенно у грузовых автомобилей, седельных тягачей и автобусов. Кроме того, при торможении по мере увеличения замедления вертикальные реакции на передних колесах возрастают, а на задних колесах уменьшаются. Для повышения эффективности торможения тормозные силы должны меняться соответственно изменениям вертикальных реакций на передних и задних колесах.  [c.248]

Независимо от того, стойт ли автомобиль на месте или движется,. на него всегда действуют определенные силы. Если он неподвижен и установлен на горизонтальной площадке, на него действует только сила тяжести (вес автомобиля), направленная вертикально вниз, и силы противодействия дороги давлению колес (реакция дороги), направленные в противоположную сторону. На автомобиле, стоящем на наклонной плоскости, сила тяжести раскладывается на две составляющие, одна из которых прижимает автомобиль к дороге, а другая стремится его опрокинуть в поперечной или продольной плоскости — в зависимости от направления уклона дороги. При. этом опрокидывающий момент будет тем больше, чем больше угол наклона автомобиля и выше его центр тяжести. На автомобиль, находящийся в движении (рис. 96), кроме силы тяжести, действуют тяговая сила, сила сопротивления качению, сила сопротивления воздуха, сила сопротивления подъему (при движении в гору), сила сопротивления боковому скольжению. При повороте автомобиля на него действуют центробежная сила, а при разгоне (торможении) — сила инерции.  [c.163]


Внешние силы, действующие на машину » Ремонт Строительство Интерьер


В общем случае движения на лесную машину действуют силы сопротивления: качению — Pf, подъему — Pi, инерции — Pj, воздушной среды — Pw, перемещению прицепов или волочащихся деревьев — Pкр.
Сила сопротивления качению возникает на поверхности контакта колеса или гусеницы с полотном пути.
При качении колеса вследствие деформации опорной поверхности и шины возникает сила сопротивления качению. При расположении жесткого колеса на деформированной поверхности (рис. 14.1а) происходит углубление. При этом вес, действующий на колеса, и реакция опорной поверхности совпадают по вектору и уравновешивают друг друга. В процессе качения колесо подминает под себя грунт и позади него остается след — колея, рис. 14.1б. Равнодействующая реакций грунта Z смещена с вертикальной оси и вынесена вперед на некоторое расстояние а. Тогда сила Т, возникающая под действием крутящего момента Мк, и возникающая касательная реакция опорной поверхности образуют момент силы Pf*b. Тогда можно заменить:

Принимая b = zd, получаем

Отношение — в теории машин называют коэффициентом сопротивления качению f.
Или:

где rd — динамический радиус колеса.
При движении машины по горизонтальному участку пути сопротивление качению колеса Pf определяется по формуле:

Сила сопротивления качению колесной машины равна:

где Z — нормальные опорные реакции на оси машины; f — коэффициент сопротивления качению колес машины; G — вес груженой машины.

Силы сопротивления качению гусеничного трактора суммируются из сил преодоления трения в движителе и силы затрачиваемых на нормальную к опорной поверхности деформацию почво-грунта (рис. 14.2):

где T — горизонтальная составляющая равнодействующей реакции почвы на лобовой участок гусениц при движении по деформирующейся поверхности; Mrк — момент от сил трения опорных катков и в элементах звеньев гусеницы.

Сила сопротивления качению гусеничного трактора определяется по аналогии с колесной машиной:

где G — вес трактора с грузом.
Сила сопротивления движению на подъем возникает при преодолении машиной подъемов дороги или волока, которые характеризуются углом подъема а или величиной подъема i (в промилле). Величина подъема определяется отношением превышения H к заложению S (рис. 14.3).
Для определения силы сопротивления движению на подъем вес машины G можно разложить на две составляющие: касательную к поверхности пути

и нормальную составляющую

Лесовозные автопоезда в основном эксплуатируются на дорогах, где углы подъемов не превышают 10°, а трелевочные тракторы — при α

Pi=Gi,


где i — подъем, %.

Уменьшение нормальной составляющей веса машины при ее движении на подъем отражается на силе сопротивления качению, которую в этом случае определяют по уравнению:

Сила сопротивления дороги. Коэффициент сопротивления качению f и уклон i определяют свойства дороги. Поэтому в теории движения колесных и гусеничных машин введено понятие сопротивления дороги Pψ:

где ψ — коэффициент сопротивления дороги.
Сила инерции возникает в процессе разгона масс, совершающих поступательное и вращательное движения. Обозначив через G полный вес машины, найдем силу инерции массы, совершающей поступательное движение:

где j — ускорение машины; g — ускорение свободного падения.
Влияние вращающихся масс на силу инерции машины определено по формулам приведения моментов их инерции к поступательной скорости движения, но в студенческих расчетах можно учитывать коэффициент учета вращающихся масс δ.
Тогда сила инерции определяется так:

Коэффициент δ, определяется по эмпирическим формулам
δв = 1,04 + 0,05 i2тр — для автомобиля;
δв = 1,2 + 0,002 i2тр — для гусеничных тракторов,
где iтр — передаточное число трансмиссии.
Сила сопротивления воздушной среды, или сила аэродинамического сопротивления, начинает существенно препятствовать движению машины на скоростях более 20…30 км/ч. Сила сопротивления воздушной среды определяется по формуле:

где k — коэффициент обтекаемости транспортной системы, H*с/м4; F — площадь проекции лобовой поверхности машины на плоскость перпендикулярную полотну пути (лобовая поверхность), м2; v — скорость машины, км/ч.
Значения коэффициента обтекаемости для современных грузовых автомобилей и автопоездов находятся в пределах 0,6…1,0. Однако стволы деревьев и необрубленные сучья создают дополнительные аэродинамические сопротивления и, по данным исследований для лесовозных автопоездов К = 2,8…3,0 H*с/м4. При выполнении расчетов площадь лобовой поверхности определяют по приближенной формуле:

F = 0,9ВН,


где В — колея машины; H — наибольшая высота машины с учетом высоты перевозимого груза.
Сила сопротивления движению прицепа или волочению пачки деревьев действует на лесовозные автопоезда и трелевочные тракторы. Сила тяги на крюке Ркр этих машин имеет разную природу.
Сила, необходимая для буксировки прицепа или полуприцепа-роспуска, равняется сумме всех сопротивлений их перемещению. В общем случае этими сопротивлениями будут: сопротивления качению колес, сопротивления движению на подъем, сопротивление воздушной среды и инерции:

где Gпр — вес прицепа или роспуска; Q2 — нагрузка, приходящаяся на прицеп или роспуск; δ’в — коэффициент учета вращающиеся масс прицепа или роспуска; Pw пр — сила сопротивления воздушной среды, действующая на прицеп или полуприцеп-роспуск. При расчете силу сопротивления воздушной среды прицепа принимают около 20-25% от силы Рw, а для прицепа-роспуска равной нулю.
Трелевка леса в полупогруженном положении, когда часть веса пачки Q1 находится на тракторе, а остальная часть Q2 волочится по земле. При небольших скоростях движения трелевочных тракторов сопротивлением воздушной среды можно пренебрегать и считать, что сила сопротивления на крюке у трелевочных тракторов соответствует силе сопротивления волочению древесины по земле. Тогда:

где fп — коэффициент сопротивления волочению пачки древесины; Q2 — вес волочащейся части пачки.
Для преодоления сил сопротивления движению к ведущим колесам машин от двигателя через трансмиссию подводится крутящий момент. Под действием крутящего момента колеса или гусеницы стремятся сдвинуть грунт назад, и на поверхности контакта возникает касательная реакция Рк, которая равняется силе, вызывающей движение машины.
Касательная реакция грунта действует на ведущие колеса или гусеницы и препятствует вращению колес и перематыванию гусеничных лент. На радиусе ведущего колеса rд касательная реакция грунта создает момент сопротивления:

Крутящий момент Mк обычно во много раз превышает крутящий момент двигателей внутреннего сгорания. Поэтому в трансмиссиях лесных автомобилей и тракторов предусматриваются значительные передаточные числа iтр, увеличивающие крутящий момент двигателя Mе. При передаче крутящего момента через трансмиссию и гусеничный движитель часть энергии переходит в энергию трения в зацеплениях шестерен, в подшипниках, перемешивания масла, качения опорных катков по беговым дорожкам гусениц и т. д. Потери энергии в трансмиссии и гусеничном движителе учитываются соответствующими КПД — ηтр, ηг. С учетом этих потерь и передаточного числа трансмиссии крутящий момент на коленчатом валу двигателя при равномерном движении машины:

или для колесной машины:

для гусеничной машины:

Касательная реакция грунта является суммой сопротивлений движению машины, которые определяют возможность движения конкретной машины в тех или иных условиях. Поэтому при выборе параметров двигателя и трансмиссии лесной машины приходится анализировать баланс действующих на нее внешних и внутренних сил.

Силы, действующие на автомобиль

В первых двух статьях нашего цикла мы много говорили о скользкой дороге и всяких неприятностях, с этим связанных. Однако, еще в самом начале мы упоминали о том, что «зимняя» дорога совсем не так кардинально отличается от «летней», как это может показаться на первый взгляд.

Дорога – не место для ошибок

Действительно, ведь автомобиль, как физический объект, и законы природы, которым он подчиняется, остаются неизменными в любое время года. Поэтому и не может существовать принципиального различия между поведением водителя на сухой дороге и на скользкой. Другое дело, что более тяжелые условия движения, как, например, зимой, ярче выявляют те ошибки, которые были всегда. Сухая дорога и умеренные скорости способны простить и сгладить многие погрешности водителя, даже грубые. И только Ее Величество Зима во всей красе показывает нам истинное положение наших водительских дел и ставит все на свои места. Скажем ей за это спасибо!

Кстати, обращали ли вы внимание на то, что подавляющее число великих раллистов мира, многократных чемпионов, выходцы из Финляндии? Это страна, которая почти не знает чистого, сухого асфальта! Это люди, которые с самого начала своей водительской и спортивной карьеры были в тяжелых условиях. Северная зима показала им, что же такое на самом деле вождение автомобиля.

Мы также можем гордиться – советская раллийная школа занимала видное место в мире. Так, что если гололед повергает вас в ужас, не стоит отчаиваться и ставить автомобиль в гараж до весны – просто пришло время всерьез задуматься над тем, как грамотно водить автомобиль.

Начнем с понимания законов физики

Предыдущие наши статьи преследовали цель, так сказать, оказать первую неотложную помощь автомобилистам, «пострадавшим» от неожиданного прихода зимы. Теперь же мы возвращаемся к корням. В первом разделе мы уже упоминали о том, что автомобиль – это тяжелый физический объект (более тонны). С любой массой нет никаких проблем до тех пор, пока она двигается прямолинейно и равномерно – вспоминайте школьную физику. Как только вы предпринимаете попытку разогнать, замедлить или повернуть тонну железа, сразу возникают трудности – сила инерции, которая изо всех сил сопротивляется вашим желаниям.

Возможность реализации любых ваших водительских замыслов практически полностью зависит от того, на что способны ваши колеса – другими частями автомобиль за дорогу не цепляется. Поэтому, для того, чтобы грамотно управлять машиной, водителю требуется четко понимать, какую работу выполняют колеса его автомобиля, и что от них можно требовать.

Колеса имеют определенный потенциал сцепления с дорогой. Этот потенциал зависит от множества параметров:

  • масса автомобиля;
  • размеры колеса;
  • давление в шине;
  • состояние шины;
  • состояние дорожного покрытия;
  • температура и т.д.

Вдаваться в глубокие технически подробности мы не будем. Просто скажем, что сцепление колеса с дорогой меняется, но в каждый конкретный момент оно имеет какую-то величину.

Предположим, что в данных условиях колесо вашего автомобиля способно выдержать силу в 10 баллов. Т.е., если сила превышает 10 баллов, колесо начинает скользить. Эту силу колесо способно выдержать, как в продольном, так и в поперечном направлении. В продольном направлении – разгон, торможение; в поперечном – поворот. Если разгон настолько интенсивный, что тяговая сила от двигателя превышает 10 баллов – колесо буксует (интенсивность разгона падает). Если торможение настолько интенсивное, что тормозная сила превышает 10 баллов – колесо блокируется (интенсивность торможения падает). Точно так же в повороте – если поворот настолько интенсивный, что боковая сила превышает 10 баллов, автомобиль соскальзывает с дороги.

Важно помнить о том, что сумма всех сил, действующих на колесо, не должна превышать наши условные 10 баллов. И если водитель начал тормозить, то он уже отнял определенную величину потенциала сцепления колеса с дорогой на торможение, соответственно, на поворот возможностей осталось меньше. Одним словом, чем интенсивнее вы разгоняетесь или тормозите, тем меньше у вас остается возможности поворачивать – колеса тратят почти весь свой потенциал сцепления с дорогой на разгон или торможение, и на поворот у них не осталось больше сил.

Понимание изложенного принципа непосредственно влияет на вашу каждодневную езду в автомобиле. Максимально интенсивный разгон, так же как и максимально интенсивное торможение, возможны только на прямой. Понятно почему – колеса тратят последние силы на то, чтобы разгонять или тормозить автомобиль, и если в этот момент вы начнете поворачивать, добавляется боковая сила, которая играет роль последней капли в переполненной чаше – автомобиль начинает соскальзывать с дороги.

Справедлив и обратный случай – в интенсивном повороте не осталось возможности тормозить или разгоняться. Мы вплотную подошли к пониманию наиболее рационального, грамотного и безопасного стиля езды.

Воспользуйтесь советами профессионалов

Старайтесь завершить торможение до поворота. Это позволит вам двигаться в самом повороте с ровным газом – ничто не будет мешать колесам «сосредоточиться» на борьбе с боковыми силами. И на выходе из поворота, по мере возврата рулевого колеса в нейтральное положение, вы сможете все больше и больше ускоряться.

Запомните это золотое правило прохождения любых поворотов – поменьше скорость на входе (в начале поворота), зато побольше на выходе (в конце поворота). Даже если вы очень спешите, не поддавайтесь искушению влететь в поворот на полном ходу. Такая ошибка приведет к тому, что вы будете интенсивно тормозить на дуге поворота, а это крайне нежелательно, т.к. вы заставите колеса вашего автомобиля буквально «разрываться» на два фронта – торможение и поворот.

В результате, можно просто превысить возможности колес цепляться за дорогу и вылететь с нее на обочину, соседнюю полосу, встречную… Даже если вам удастся остаться на дороге, все равно весь поворот вы будете бороться с автомобилем, и в конце выедете с дрожащими руками на минимальной скорости. Страшно, опасно и скорость прохождения поворота маленькая.

Намного рациональнее перед поворотом затормозить до заведомо умеренной скорости (в зависимости от условий движения), аккуратно и спокойно направить автомобиль по дуге, убедиться в том, что ваши расчеты верны, и автомобиль превосходно держится за дорогу, а вот тогда, на выходе из поворота, когда колеса возвращаются в прямолинейное положение, давите на газ сколько угодно. Надежно, безопасно, комфортно, и максимально быстро!

Краткая информация для активных водителей – первым приезжает не тот, кто пережил больше страха, создал много визга и чудом остался на дороге, а тот, кто думал и действовал спокойно и мудро.

Думайте и готовьтесь к поворотам заранее. Поворот – это целый комплекс мер: вам нужно успеть снизить скорость, выбрать более подходящую передачу, увидеть оптимальную траекторию движения, подхватить руль поудобнее и только тогда поворачивать.

На всех указанных элементах мы еще остановимся подробнее в дальнейшем. Тем не менее, понятно, что некая подготовка к повороту имеется всегда. Начинайте ее заранее – лучше подготовиться раньше и потом плавно разгоняться, чем опоздать и, свалив все действия в кучу, судорожно пытаться что-то предпринять в последний момент. Даже если каждое из действий отнимает у вас только мгновение, их сумма представляет собой пусть короткое, но время.

Заложите это время в свой план, чтобы успеть до поворота.

Оценивайте ситуацию и действуйте на опережение

Мы заговорили о планировании своих действий заранее. Это одно из основных умений профессионала – это то, что отличает грамотного водителя от неграмотного. Многие начинающие водители думают, что мастерство профессионала заключается в умении мгновенно выполнять какие-то действия. Это не так.

Неопытный водитель, как правило, смотрит «себе под капот» и все время решает текущие задачи, буквально натыкаясь на них каждый раз. Опытный водитель смотрит далеко вперед и поэтому он может позволить себе действовать спокойно, т.к. он все видит заранее и может спокойно подготовиться.

Именно отсутствие необходимости в резких, экстренных действиях – главный гарант надежного, безопасного и быстрого движения. Поступайте предусмотрительно и мудро за рулем, результат будет превосходным – удовольствие без последствий!

Результирующая сила — урок. Физика, 7 класс.

Если на тело одновременно действует несколько сил, тогда состояние тела или его движение определяет результирующая сила — сумма всех сил.

Если силы действуют в одном направлении, результирующая сила равна сумме сил.

F1→F2→ 

 

Рис. \(1\). Силы в одном направлении

 

Fрез=F1+F2

 

Рис. \(2\). Сила тяги обеих лошадей суммируется

 

Обрати внимание!

Если силы действуют в противоположных направлениях, результирующая сила равна разности сил — от большей силы отнимают меньшую, результирующая сила действует в направлении большей силы.

Например, при движении автомобиля на него действуют две силы: сила тяги \(Fт\), создаваемая двигателем и направленная в сторону движения, и сила трения \(Fтр\) с поверхностью дороги, которая направлена в противоположном направлении (силу сопротивления воздуха не учитываем).   

 

Fрез=Fт−Fтр

 

 

Рис. \(3\). Результирующая сила

 

Fрез=Fтр−Fт

 

В этом случае движение автомобиля будет замедленным, так как сила трения больше, чем сила тяги.  

 

 

 

 

 

Если сила тяги одинакова с силой трения, тогда автомобиль двигается с постоянной скоростью.

Эти силы компенсируют друг друга. Сумма всех сил равна нулю, и автомобиль двигается без ускорения.

 

 

Рис. \(4\). Силы, действующие на автомобиль

 

Fрез=0

1-й закон Ньютона:

Если на тело не действуют силы или они уравновешены, тогда тело остаётся неподвижным или двигается равномерно и прямолинейно.

Когда автомобиль стоит на дороге, он находится в статическом равновесии. На него действуют две силы — сила притяжения (гравитации) и сила упругоcти опоры (дороги), которые компенсируют друг друга, их сумма равна нулю.


Каждое тело стремится оказывать сопротивление изменению скорости, оказывая сопротивление силе, вызывающей ускорение.

Стремление тела сопротивляться изменению скорости называют инерцией, или инертностью. Масса является мерой инертности (чем больше масса, тем больше инертность тела). 

 

Обрати внимание!

Из-за инерции тормозной путь у автобуса длиннее, чем у легкового автомобиля!

Источники:

Рис. 3. Результирующая сила. © ЯКласс.

 

Естественные силы, трение, тяга и баланс

Все в известной вселенной подвержено воздействию естественных сил, таких как инерция, гравитация, трение и энергия. Ваш автомобиль не является исключением, он полагается на законы физики в своей работе. Без естественных сил ваш автомобиль не смог бы заводиться, двигаться, останавливаться или менять направление. То, как ваше транспортное средство взаимодействует с этими силами, в некоторой степени определяется его конструкцией. Поскольку законы физики постоянны и неизменны, производители автомобилей могут использовать их для создания стабильных, безопасных и эффективных транспортных средств.

Даже больше, чем дизайн, поведение вашего автомобиля в ответ на невидимые, естественные силы, действующие на него, определяется тем, как вы его ведете . В рамках обучения водителя вы должны узнать, как различные силы и природные законы влияют на ваш автомобиль, чтобы сохранять контроль и надлежащим образом реагировать в чрезвычайных ситуациях.

Это может показаться огромным мероприятием, но помните, что вы уже опытный водитель другого, совершенно другого транспортного средства, которое также находится во власти природных сил: вашего тела.Как и ваша машина, ваше тело питается энергией, ему противодействует трение, оно действует по инерции и притягивается к центру Земли под действием силы тяжести. Несмотря на постоянное вмешательство этих сил, вы можете ходить, бегать, прыгать, приседать и заниматься бесчисленным множеством других физических нагрузок. В конце концов, у вас будет такой же уровень интуитивного управления автомобилем.

Природные силы всегда действуют на ваш автомобиль, когда он неподвижен и движется. В идеальной дорожной ситуации на ровной, прямой, ровной дороге с хорошим покрытием вы не можете останавливаться, чтобы оценить влияние этих сил.В более сложных условиях вам понадобится осведомленность и понимание природных сил, чтобы принимать решения о безопасном вождении и избегать столкновений. Если вы едете по холму, по повороту, выполняете поворот или преодолеваете плохую погоду, ваше знание законов природы может легко определить разницу между поездкой без происшествий и катастрофической аварией.

Если вас немного пугает перспектива изучения законов физики в рамках курса обучения водителя — не паникуйте.Чтобы научиться управлять автомобилем, не требуется знания сложных математических уравнений или абстрактных теорий. Все, что вам нужно, — это общее представление о природных силах, воздействующих на ваш автомобиль, и о том, как вы можете работать с ними, чтобы сохранять контроль и оставаться в безопасности на дорогах. Начнем с краткого обзора тем, затронутых в этом разделе.

Сила тяжести

Наше исследование науки о вождении начинается с рассмотрения силы тяжести. Эта сила существует до некоторой степени между любыми двумя объектами во Вселенной, хотя обычно мы замечаем только гравитационное притяжение Земли.Гравитация притягивает предметы; сила, с которой он стягивает их вместе, определяется их массой и расстоянием между ними. Более крупные объекты (например, Земля) имеют более сильное гравитационное притяжение. Мы углубимся в теорию гравитации, ее историю и эффекты более подробно в следующем модуле этого раздела. На данный момент все, что вам нужно знать, это то, что сила тяжести тянет ваш автомобиль к центру земли и будет влиять на него по-разному, в зависимости от распределения веса, уклона дороги, по которой вы едете, и различных других природных сил.

Сила тяжести, направленная вниз, влияет на скорость вашего автомобиля при движении или парковке на склоне. При движении в гору ваш автомобиль будет тянуть назад под действием силы тяжести и эффективно замедлять движение. На спуске он будет тянуться вперед, что приведет к увеличению скорости. В этом разделе вы узнаете, как изменить свое поведение при вождении, чтобы компенсировать влияние силы тяжести при движении по склону.

Законы движения Ньютона

Прежде чем мы продолжим наши поиски понимания того, как природные силы влияют на движение транспортного средства, мы должны сначала взглянуть на универсальные правила, управляющие движением.Этот модуль охватывает три закона движения Ньютона, которые вместе могут предсказывать и объяснять, как любой объект движется (или не движется) в ответ на внешние силы.

Первый закон Ньютона касается инерции. Инерция — это причина того, что изменение скорости или направления движущегося объекта всегда требует больше усилий, чем поддержание их постоянными. При вождении инерция делает маневрирование транспортного средства на повороте или его остановку физически сложным, особенно если у вас нет рулевого управления с гидроусилителем или тормозов!

Его второй закон относится к силе, которую проявляет движущийся объект.Вы когда-нибудь задумывались, почему автомобиль, сталкивающийся со стеной, наносит больше повреждений на скорости 30 миль в час, чем на скорости 15 миль в час? Наше исследование силы покажет вам, как она рассчитывается и, следовательно, почему одни объекты обладают большей силой, чем другие. Третий закон Ньютона касается действия и противодействия. Это правило объясняет, почему автомобиль, наезжающий на стену, не только повреждает стену, но и сам получает повреждения. Информация, представленная в этом модуле, поможет вам понять концепции, обсуждаемые в остальной части раздела; не забудьте найти время, чтобы прочитать его!

Энергия и потенциальная энергия

Количество энергии объекта определяет его способность выполнять работу.Объекты с большей энергией могут работать больше или дольше, чем объекты с меньшей энергией. Хотя энергия бывает разных форм, в ходе этого курса мы сосредоточимся на энергии движения или кинетической энергии. Кинетическая энергия вырабатывается движущимся объектом. У неподвижных объектов все еще есть энергия, хотя она известна как потенциальная энергия. Чем выше вероятность того, что объект начнет двигаться, тем больше у него потенциальной энергии. Когда дело доходит до вождения, количество кинетической энергии вашего автомобиля будет влиять на то, насколько легко замедлить или остановиться; чем больше энергии, тем тяжелее будет.

Скорость оказывает значительное влияние на энергию и, следовательно, на то, сколько времени потребуется вашему автомобилю, чтобы полностью остановиться в аварийной ситуации. Узнайте, как энергия влияет на тормозной путь и силу удара при столкновении, в этом важном модуле.

Центробежные и центростремительные силы

Центробежные и центростремительные силы действуют противодействуя друг другу и воздействуют на объекты, движущиеся по криволинейной траектории. Если вы не проводите все свое время по прямым скоростным автомагистралям, эти силы будут регулярно влиять на поведение вашего автомобиля на дороге.Чтобы понять центробежные и центростремительные силы, полезно представить любую кривую, по которой вы едете, как часть полного круга. Центростремительная сила тянет ваш автомобиль к центру этого круга, в то время как центробежная сила тянет его прямо от центра. Во время движения по извилистой дороге на вашу машину всегда будут действовать обе эти силы. Вы должны научиться находить баланс между ними, чтобы избежать недостаточной поворачиваемости (недостаточной поворачиваемости) и избыточной поворачиваемости (слишком большой поворачиваемости) при движении по изогнутой дороге.Здесь подробно описаны навыки и методы, необходимые для этого.

Трение и ваши шины

Далее мы рассмотрим силы сопротивления, которые влияют на внутренние движущиеся компоненты вашего автомобиля, и то, как его шины сцепляются с поверхностью дороги. Центральное место в этой дискуссии занимает трение. Трение описывает сопротивление между любыми двумя контактирующими поверхностями, когда они скользят друг относительно друга. Низкое трение обеспечивает легкость движения, а высокое трение затрудняет движение.Трение, возникающее между автомобильными шинами и дорогой, называется тягой.

Трение между шинами вашего автомобиля и дорогой должно быть высоким, чтобы автомобиль мог двигаться, а вы могли эффективно управлять. Без трения шины будут скользить по поверхности асфальта, а не цепляться за него и катиться по нему. Автомобильные шины и дорожное покрытие предназначены для максимального увеличения трения, но вы также можете многое сделать во время вождения, чтобы улучшить сцепление с дорогой или восстановить трение, если ваши колеса начали буксовать.Узнайте о трении, почему оно так важно и что вы можете сделать, чтобы его удержать, в этой части раздела.

Здесь мы также имеем дело с понятием, известным как сопротивление качению. Сопротивление качению означает, насколько ваши шины сжимаются при катании по дороге, и в значительной степени определяется давлением в шинах. Это важный вопрос, с которым нужно разобраться, поскольку чрезмерное сопротивление качению может привести к перегреву и, в конечном итоге, к выходу из строя шины.

Трение и торможение

Помимо связи между шинами и дорогой, трение играет роль в различных других важных системах транспортного средства.Пожалуй, самое главное из них — это тормоза! Когда вы нажимаете педаль тормоза, колодки прижимаются к вращающимся колесам, создавая трение, которое замедлится и в конечном итоге остановит автомобиль. Трение в конечном итоге приведет к износу, поэтому так важно соблюдать правильную технику торможения. Все тормозные системы со временем изнашиваются, но это произойдет гораздо раньше, чем следовало бы, если вы будете использовать тормоза во время движения.

Правильное торможение важно как для вашей безопасности, так и для здоровья вашего автомобиля.Чтобы замедлить или остановиться, тормоза должны поглощать кинетическую энергию автомобиля. Для этого может не хватить времени, если вы резко нажмете ногой на тормоз. Когда энергия движения транспортного средства не будет полностью поглощена, колеса заблокируются, и вы продолжите движение вперед. Если вы не водите автомобиль с АБС, постепенное торможение является ключевым моментом.

Что влияет на тягу?

На этом этапе этого раздела вы узнаете о трении и той жизненно важной роли, которую оно играет в обеспечении сцепления шин с поверхностью дороги.Это сопротивление между шиной и асфальтом известно как сцепление. Обеспечение хорошего сцепления с дорогой является основной задачей при проектировании транспортных средств, шин и проезжей части. Однако это не единственные факторы, влияющие на ваше сцепление с дорогой. Другие включают:

  • Погодные условия
  • Распределение веса вашего автомобиля
  • Уклон дороги (подъем или спуск)
  • Материал покрытия проезжей части
  • Предстоящие маневры, такие как повороты, торможение и ускорение

В этом модуле вы узнаете, как настроить свое поведение при вождении в соответствии с дорожными условиями и сохранить сцепление с дорогой.

Как исправить потерю тяги

Несмотря на все ваши усилия, однажды вы можете оказаться в пугающей ситуации, когда ваши шины потеряют сцепление с дорогой и начнут скользить по поверхности дороги. Ключ к восстановлению контроля, когда это происходит, — сохранять спокойствие и применять методы коррекции тяги, описанные в этом модуле.

При потере тяги передних колес вашего автомобиля рулевое управление может стать совершенно неэффективным. На повороте или повороте это приведет к явлению, известному как недостаточная поворачиваемость.Противоположный эффект, известный как избыточная поворачиваемость, возникает при потере сцепления задних колес при движении по повороту дороги. Обе ситуации могут привести к серьезным сбоям при неправильном обращении. Избыточная поворачиваемость может вызвать пробуксовку задних колес и съехать с дороги или выехать на полосу встречного движения. Когда возникает недостаточная поворачиваемость, водители могут выезжать на обочину дороги. Если вы когда-нибудь столкнетесь с какой-либо из этих ситуаций, напоминание информации, изложенной в этом модуле, может спасти вам жизнь.

Балансировка автомобиля

Следующие три модуля этого раздела посвящены распределению веса на четыре колеса вашего автомобиля и тому, почему это должно иметь значение для вас как водителя. Идеальным балансом транспортного средства было бы равное количество веса, прижимающего каждую шину к поверхности дороги. Когда вес смещается вперед, назад, влево или вправо от транспортного средства, шины, несущие меньший вес, будут страдать от снижения тяги. Состояние вашего автомобиля, любые маневры, которые вы выполняете, регулировка скорости и силы тяжести — все это определяет, как вес распределяется по колесам автомобиля.Мы полностью исследуем эти идеи в этом первом модуле баланса транспортных средств. Вот краткое изложение, чтобы вы начали:

  • При ускорении вес переносится на задние колеса.
  • При торможении вес переносится на передние колеса.
  • При движении в гору вес переносится на задние колеса.
  • При движении под гору вес переносится на передние колеса.
  • При повороте влево нагрузка переносится на правые колеса.
  • При повороте вправо вес переносится на левые шины.

По мере того, как вы чувствуете себя более комфортно за рулем, вы научитесь ощущать сдвиги в балансе вашего автомобиля при ускорении, замедлении или выполнении маневров. В конечном итоге ваша цель будет заключаться в максимальном увеличении сцепления с дорогой за счет корректировки вашего поведения при вождении, чтобы автомобиль оставался максимально сбалансированным.

Поддержание баланса автомобиля

В нашем следующем модуле рассматриваются четыре типа дисбаланса веса, которые могут возникнуть в движущемся транспортном средстве. Это:

  1. Наклон вперед: вес движется к передней части автомобиля.
  2. Наклон назад: вес перемещается в сторону задней части автомобиля.
  3. Крен: вес, перемещающийся с одной стороны транспортного средства на другую.
  4. Рыскание: вес, вращающийся вокруг центра тяжести автомобиля.

Часто вы будете сталкиваться с дорожными условиями, в которых одновременно возникают сдвиги веса по тангажу и крену. Поддержание контроля над вашим автомобилем потребует прогнозирования и компенсации этих сдвигов баланса с помощью правильно рассчитанных по времени корректировок скорости. Вы также должны убедиться, что ваши шины находятся в хорошем состоянии и могут сцепляться с поверхностью дороги, так как последствия дисбаланса автомобиля сложнее контролировать, когда тяга и так плохая.

Балансировка автомобиля в сложных условиях

Изменения баланса вашего автомобиля легко прогнозировать и управлять ими, когда вы едете в идеальных условиях по прямой и ровной дороге. В сложных дорожных условиях поддержание баланса транспортного средства является немного более сложной задачей. Как мы уже обсуждали, центр тяжести транспортного средства (и, следовательно, его распределение веса) зависит от уклона и формы дороги, поверхности дороги, погодных условий и других факторов окружающей среды.

Этот модуль проведет вас через типичные сценарии вождения, научит, как компенсировать изменение веса и оптимизировать тягу. Секрет успешного выполнения корректировки веса заключается в том, чтобы дать себе достаточно времени, чтобы спланировать и выполнить их. Как защитный водитель, вы должны постоянно сканировать проезжую часть впереди, чтобы искать изменения уклона, дорожного покрытия и поворотов дороги, которые повлияют на баланс вашего автомобиля.

Физика столкновений

Наш научный модуль завершается рассмотрением того, как природные силы взаимодействуют и определяют серьезность дорожно-транспортных происшествий.Вот что вам нужно знать:

  • Количество кинетической энергии объекта определяет силу, которую он проявляет и поддерживает во время столкновения.
  • При ударе о неподвижный объект серьезность столкновения будет зависеть от того, насколько быстро вы путешествуете и насколько хорошо объект, с которым вы сталкиваетесь, может поглотить вашу кинетическую энергию.
  • При лобовом столкновении двух транспортных средств более тяжелые и быстро движущиеся машины всегда наносят больший урон.

Когда водитель понимает, как энергия и сила влияют на серьезность дорожно-транспортных происшествий, он может предпринять шаги, чтобы минимизировать серьезность при столкновении с неминуемой аварией.Узнайте, как защитить себя в чрезвычайной ситуации, и познакомьтесь со встроенными функциями безопасности вашего автомобиля.

Работа с силами природы

Правила, управляющие движением в нашей Вселенной, нельзя изменить, согнуть или нарушить. Гравитация, сопротивление, инерция, энергия, сила и другие концепции, обсуждаемые в этом модуле, постоянны и, как таковые, равны предсказуемым . Понимая, как движется ваш автомобиль и какие силы на него действуют, вы можете работать с силами природы, чтобы добиться от своего автомобиля максимальной производительности.Не соблюдайте эти законы, и управление автомобилем всегда будет сложной задачей — иногда в буквальном смысле! На этой ноте давайте начнем с нашей первой важной концепции: гравитации.

Вы бы сдали экзамен по вождению сегодня?

Узнайте с помощью нашей бесплатной викторины!

ПРОЙТИ БЕСПЛАТНУЮ ТЕСТ
Понравилась статья? Ставьте нам 5 баллов!

Щелкните звездочку, чтобы добавить свой голос

5.0 из 5 звезды на основе 6 голосов.

Разгон и торможение

2D модель твердого тела

Представьте себе автомобиль, стоящий на земле, как показано на рисунке. ниже.Мы возьмем всю машину с колесами как одиночное твердое тело. Ясно, что это неточно (колеса не может повернуться), но он все еще полезный.

Начнем с неподвижной машины, сидящей на Дорога. Гравитация действует вниз через центр масс, в то время как на колеса действуют силы реакции вверх и соответствующие равные и противоположные силы направлены вниз на земля. Поскольку автомобиль не ускоряется, общая силы на машине уравновешены, как мы видим на свободное тело диаграмма.

Когда водитель нажимает на газ педали, это приводит к тому, что автомобиль толкает назад дорога, создавая чистую поступательную силу на ведущие колеса ( задние колеса для нашей машины), и машина разгоняется до крейсерская скорость. Здесь мы включаем воздух сопротивление, но пренебрежение качением сопротивление, а при движении с постоянной скоростью движущая сила точно уравновешивает силу сопротивления воздуха сопротивление.Когда водитель нажимает на педаль тормоза, машина толкает дорогу, чтобы замедлить вниз, давая обратные силы на оба колеса и вызывая автомобиль должен замедлиться до остановки.

ускорять тормозить диаграмма свободного тела компоненты анимация

Повторите ускорение / торможение выполнить цикл несколько раз, показывая свободное тело диаграмма. Обратите внимание на горизонтальные и вертикальные силы. дороги на колесах автомобиля.Также просмотрите силы как в вектор компоненты и как векторы полной силы.

Вертикальные силы дороги на автомобиль всегда должны уравновесить гравитационную силу, но мы видим, что распределение между передними и задними колесами изменяется по мере изменения автомобиль ускоряется и замедляется. Это потому, что горизонтальные движущие и тормозные силы ниже центра массы и произвести момент.Автомобиль не вращается, так что этому моменту надо противодействовать землей силы. Направления силы означают, что задние колеса принимают больший вес при разгоне, в то время как передние колеса принимают больший вес при торможении.

Сил, действующих на автомобиль при движении. Сборник вопросов и задач по физике — Лукашик В.И. Главное

На движущуюся машину действует ряд сил, часть которых направлена ​​вдоль оси движения машины, а деталь расположена под углом к ​​этой оси.Мы согласны называть первую из этих сил продольной, а вторую — боковыми.

Рис. Схема сил, действующих на ведущее колесо.
А — состояние неподвижности; б — условие движения

Продольная сила Может быть направлена ​​как вдоль, так и против движения автомобиля. Силы, направленные по ходу движения, движутся и стремятся продолжить движение. Силы, направленные против движения движения, являются силами сопротивления и стремятся остановить машину.

На автомобиль, движущийся по горизонтальному и прямому участку дороги, действуют следующие продольные силы:

  • истинная сила
  • Прочность сопротивления воздуха
  • сила сопротивления качению

При движении автомобиля в горах возникает сила сопротивления, а при ускорении автомобиля — силовое сопротивление (сила инерции).

Истинная сила

Крутящий момент, развиваемый автомобилем, передается на ведущие колеса.В передаче крутящего момента от двигателя к ведущим колесам задействованы передаточные механизмы. Крутящий момент на ведущих колесах зависит от крутящего момента двигателя и передаточных чисел коробки передач и главной передачи. В точке соприкосновения колес с поверхностью дороги крутящий момент вызывает окружную силу. Противодействие дороге этой окружной силе выражается реактивной силой, передаваемой от дороги на ведущее колесо. Эта сила направлена ​​навстречу движению автомобиля и называется толчком или тягой.Сила тяги от колес передается на ведущий мост, а затем на раму, заставляя автомобиль двигаться. Величина тягового усилия тем больше, чем больше крутящий момент двигателя и передаточные числа коробки передач и главной передачи. Сила тяги на ведущих колесах достигает наибольшего значения при движении автомобиля на самой низкой передаче, поэтому пониженная передача используется при трогании с места автомобиля с грузом, когда автомобиль движется по бездорожью. Величина тягового усилия на ведущих колесах автомобиля ограничивается сцеплением шин с поверхностью дороги.

Силовое сцепление с дорогой

Трение, возникающее между ведущими колесами автомобиля и дорогостоящим, называется натягом сцепления. Сила сцепления равна работе коэффициента сцепления на массу сцепления, т. Е. На вес, приходящийся на ведущее колесо автомобиля. Величина коэффициента сцепления шины зависит от качества и состояния дорожного покрытия, формы и состояния рисунка протектора шины, давления воздуха в автобусе.

В легковых автомобилях полная масса распределяется по осям примерно поровну.Поэтому массу сцепки можно принять равной 50% от общей массы. В грузовых автомобилях при полной загрузке сцепки вес (вес, приходящийся на заднюю ось) составляет около 60-70% от общего веса.

Величина коэффициента сцепления имеет большое значение для работы автомобиля и безопасности движения, так как от этого зависит грузоподъемность автомобиля, качество торможения, способность, пробуксовка и движение ведущих колес. При небольшом коэффициенте сцепления касание автомобиля сопровождается пробуксовкой, а торможение — скольжением колес.В результате машину иногда не удается задеть с места, а при торможении происходит резкое увеличение тормозного пути и возникновение заноса.

На асфальтобетонных покрытиях в жаркую погоду на поверхности остается битум, из-за чего дорога становится маслянистой и более скользкой, что снижает коэффициент сцепления. Коэффициент сцепления особенно снижается, когда дорога намокает после первого дождя, когда еще нет смытой пленки жидкой грязью. Снежная или обледенелая дорога особенно опасна в теплую погоду, когда поверхность постукивает.

С увеличением скорости движения коэффициент сцепления уменьшается, особенно на мокрой дороге, так как выступы рисунка протектора шины не успевают смыть пленку влаги.

Хорошее состояние рисунка протектора шины имеет большое значение при движении по грунтовым дорогам, снегу, песку, а также по дорогам с твердым покрытием, по покрытой пленкой грязи или воды. Из-за наличия выступов площадь отсчета уменьшается и, следовательно, удельное давление на поверхность дороги увеличивается.Это позволяет легче пробивать грязевую пленку и восстанавливать контакт с дорожным покрытием, а на легком грунте происходит прямое зацепление выступов изображения почвы.

Повышенное давление воздуха в шине уменьшает ее опорную поверхность, в результате чего удельное давление увеличивается настолько, что при движении с места и при торможении может произойти разрушение резины и сцепление колес с дорогой уменьшается.

Таким образом, величина коэффициента сцепления зависит от многих условий и может варьироваться в довольно значительных пределах.Поскольку многие дорожно-транспортные происшествия происходят из-за плохого сцепления, водители должны иметь возможность приблизительно оценить величину коэффициента сцепления и выбрать скорость и скорость управления в соответствии с ней.

Прочность на воздушное сопротивление

При движении автомобиль преодолевает сопротивление воздуха, которое складывается из нескольких сопротивлений:

  • сопротивления лобового стекла (около 55-60% от общего сопротивления воздуха)
  • образовано выступающими частями — ступеньками автобуса или автомобиля, крыльями (12-18%)
  • , возникающие при прохождении воздуха через радиатор и подкарборное пространство (10-15%) и другие.

Передняя часть автомобиля воздух сжимается и поднимается, в то время как в задней части автомобиля создается разрежение, которое вызывает образование скручиваний.

Сила сопротивления воздуха зависит от величины лобовой поверхности автомобиля, его формы, а также от скорости движения. Фронтальная площадь грузового вагона определяется как расстояние между колесами (расстояние между шинами) по высоте автомобиля. Сила сопротивления воздуха увеличивается пропорционально квадрату скорости движения автомобиля (при увеличении скорости в 2 раза сопротивление воздуха увеличивается в 4 раза).

Для улучшения обтекаемости и снижения сопротивления воздуха лобовое стекло автомобиля наклонено, а выступающие части (фары, крылья, ручки дверей) установлены заподлицо с внешними контурами кузова. В грузовых автомобилях можно уменьшить мощность сопротивления воздуха, закрыв запястный поток площадкой, натянутой между крышей кабины и задней стороной.

Сила сопротивления качению

На каждое колесо автомобиля постоянно действует вертикальная нагрузка, которая вызывает вертикальную реакцию дороги.Когда машина движется по ней, сила сопротивления качению, возникающая из-за деформации шин и дороги и поворота шин по дороге.

Сила сопротивления качению равна произведению общей массы автомобиля на коэффициент сопротивления шин, который зависит от давления воздуха в шинах и качества дорожного покрытия. Вот некоторые значения коэффициента сопротивления качению шины:

  • для асфальтобетонного покрытия — 0,014-0.020
  • для гравийного покрытия-0,02-0,025
  • для песка-0,1-0,3

Повышение сопротивления

Дорога состоит из чередующихся подъемов и спусков и редко имеет длинные длинные горизонтальные участки.

При движении на подъеме автомобиль испытывает дополнительное сопротивление, которое зависит от угла дороги до горизонта. Повышение сопротивления тем больше, чем больше вес автомобиля и угол дороги. При входе на подъем необходимо правильно оценить возможности преодоления подъема.Если подъем короткий, он преодолевается с ускорением автомобиля перед подъемом. Если подъем длинный, его преодолевают на пониженной передаче, переходя на нее в начале подъема.

Когда автомобиль движется на спуске, сила сопротивления направлена ​​в сторону движения и является движущей силой.

Сила сопротивления разгону

Часть тяги для разгона расходуется на ускорение вращающихся масс, в основном маховика коленчатого вала двигателя и колес автомобиля.Чтобы автомобиль начал двигаться с определенной скоростью, ему необходимо преодолеть силу сопротивления разгону, равную произведению массы автомобиля на ускорение. Когда автомобиль ускоряется, сила сопротивления ускорению направлена ​​в сторону обратного движения. При торможении автомобиля и замедлении его движения эта сила направляется навстречу движению автомобиля. Известны случаи, когда груз или пассажиры вываливаются при резком ускорении с открытой местности, с сиденьями мотоцикла, а при резком торможении пассажиры ударяются о лобовое стекло или переднюю часть автомобиля.Для таких случаев необходимо, плавно увеличивая частоту вращения коленчатого вала двигателя, преодолевать силу сопротивления ускорению и плавно осуществлять торможение автомобиля.

Центр тяжести

На автомобиль, как и на любое другое тело, сила тяжести действует вертикально вниз. Центром тяжести автомобиля называется такая точка автомобиля, от которой вес автомобиля распределяется равномерно во всех направлениях. Центр тяжести расположен между передней и задней осью на высоте около 0.6 м для пассажиров и 0,7-1,0 м для грузовых. Чем ниже центр тяжести, тем устойчивее автомобиль против опрокидывания. При загрузке автомобиля автомобилем центр тяжести повышается в легковых автомобилях примерно на 0,3-0,4 м, а в грузовых — на 0,5 м и более в зависимости от типа груза. При неравномерной установке центр тяжести может также сместиться вперед, назад или в сторону, что нарушит сопротивление автомобиля и легкость управления.

319. Почему на Холлидице тротуар посыпался песком?
320.Почему зимой у некоторых грузовиков задние колеса обвязывают цепями?
321. Для чего, когда вы спускаетесь, одноколесную тележку с горы иногда закрепляют так, чтобы она не вращалась?
322. Почему на шинах автомобилей колесных тракторов делают глубокий рельефный рисунок (протектор)?
323. Почему осенью на трамвайных линиях, проезжающих эколо-парки, бульвары и сады, вешается предупреждающий знак «Осторожно, листопад!»?
324. Почему после дождя грунтовая дорога скользкая?
325. Почему после дождя опасно ехать на машине по грунтовой дороге под уклоном?


Фиг.79.

326. Почему некоторые мастера смазывают мыльный винт и вкручивают его в скрепляемые детали?
327. Почему скоба, на которую спускается корабль, обильно смазана?
328. Что такое насечки в куче гвоздей?
329. Назовите одну или две части изготовленного велосипеда с учетом увеличения силы трения скольжения.
330. Какого рода трение возникает при движении подвески, как показано на рисунке 78? Где сила трения о карандаш в случае А, в случае В — относительно книги?
331.Грузовая тележка перемещается (рис. 79). Какой вид Iia возникает между: а) столом и колесами; б) CKO груз; в) оси колес и корпус тележки?
332. Почему кирпичи не скатываются (рис. 80 и 81)? Какая сила удерживает их в покое? Картины, действующие на кирпичи.
333. Полоса перемещается вправо (рис. 82). Где я направлен на трение скольжения по отношению к штанге; Следующие поверхности, по которым движется штанга?
334. Лестница у стены занимает положение, изображенное на рисунке 83.Укажите направление силы трения при касании лестницы стеной и полом.


Фиг. 80.


Фиг. 81.


Фиг. 82.


Фиг. 83.


Фиг. 84.


Фиг. 85.


Фиг. 86.

335. Штанга движется равномерно (рис. 84). Где направление: а) сила упругости горизонтальной части нити; б) вертикальный; в) сила трения скольжения относительно поверхности стола относительно штанги; г) Чему равна равная сила этих сил?
336.Колесо автомобиля опущено (рис. 85). Где сила хода направлена ​​между опрокидывающим колесом и дорогим родственником: а) колесами; б) дороги? Где сила упругости дороги?
337. Книга прижата к вертикальной поверхности (рис. 86). Изобразите графически направление гравитации и трения мира, действующих на книгу.
338. Тележка равномерно движется вправо (см. Рис. 79). Какая] сила приводит к перемещению поставляемого к нему груза? Где эта сила?
339.Конвейер равномерно перемещает ящик с грузом (без проскальзывания). Когда грубая сила трения направлена ​​между конвейерной лентой и выдвижным ящиком, когда ящик: а) поднимается; б) перемещение по горизонтали; в) пропущено?


Фиг. 87.

340. Если автобус равномерно движется по горизонтальному участку пути, какова сила трения корня?
341. Парашютист, масса которого 70 кг, спускается равномерно. Какая сила сопротивления воздуха, действующая на парашютиста?
342.С помощью динамометра стержни выравниваются равномерно (см. Рис. 82). Какая сила удара между грифом и поверхностью стола? (Цена деления динамометра 1 н.)
343. Пильный диск разводится в разные стороны от плоскости пилы. На рис. 87 показаны пропеллы, изготовленные неразбавленной и разбавленной пилой. Какой пилой труднее резать: разбавленной или раздетой? Почему?
344. Приведите примеры, когда трение приносит пользу, а когда вред.

Назад 1 .. 10> ..>> Далее
333.Полоса перемещается вправо (рис. 82). Где сила трения скольжения?
334. Лестница у стены занимает положение, показанное на рисунке 83. Укажите направление силы трения в местах соприкосновения лестницы со стеной и полом.
335. Штанга движется равномерно (рис. 84). Где сила упругости нити и сила трения скольжения, возникающего при движении по поверхности стола? Что равносильно помощи этих сил?
336.Колесо автомобиля опущено (рис. 85). Где фиктивная сила скольжения между буксирующим колесом и дорогой? Сила трения дождя (эластичность дороги)?
Рис. 86.
Рис. 87.
837. Книга прижата к вертикальной поверхности (Рис. 86). Изобразите графически направление гравитации и трения мира, действующих на книгу.
338. Тележка движется равномерно (см. Рис. 79). Какая сила приводит в движение лежащий на тележке груз? Куда она нацелена?
339. Транспортер перемещает ящик с грузом (без скольжения).В каком направлении находится сила трения покоя между лентой конвейера и выдвижным ящиком?
340. Если автобус равномерно движется по горизонтальной траектории, что равно силе трения покоя?
341. Парашютист, масса которого 70 кг, движется равномерно. Какая сила сопротивления воздуха действует на парашют?
342. На динамометре штанга перемещается равномерно (см. Рис. 82). Какая сила удара между грифом и поверхностью стола? (Цена деления динамометра 1 н.)
343. Пильный диск разводится в разные стороны от плоскости пилы. На рис. 87 показаны пропеллы, изготовленные неразбавленной и разбавленной пилой. Какой пилой труднее резать: разбавленной или раздетой? Почему?
344. Приведите примеры, когда трение полезно, а когда приносит вред.
17. Давление1
345. На стол кладут два тела одинакового веса, как показано на Рисунке 88 (слева). У них такое же давление на стол? Если эти тела возьмут на себя весы, равновесие весов нарушится?
346.У нас такое же давление на карандаш, волоча его тупым и острым ножом, если прилагаемое нами одинаковое усилие?
1 При расчете принимают G = 10 H / кг.
37
347. Перемещая один и тот же груз (рис. 89), мальчики в первом случае прикладывают большую силу, чем во втором. Почему? В каком случае давление груза на пол больше? Почему?
348. Почему верхний край лопатки, на которую прижимается лапка, должен загибаться?
349. Какие режущие части косилки, соломотряса и других сельскохозяйственных машин должны быть острыми?
350.Зачем для путешествий по болотам делать настил из прутьев, бревен или досок?
351. Когда болт является болтом, к деревянным брускам под гайку и головку болта кладут широкие металлические плоские кольца — шайбы (рис. 90). Зачем это делать?
352. Почему при вытаскивании гвоздей из доски кладут железную ленту или лыжные щипцы?
353. Объясните назначение наперстка, надеваемого на палец при пришивании иглы.
354. В одних случаях давление пытаются снизить, а в других — повысить. Приведите примеры, где в технике или в повседневной жизни, а где давление увеличивается.
355. На рисунке 91 показан кирпич в трех положениях. В каком положении давление кирпича на доску будет наименьшим? Самый?
Фиг. 89.
Фиг. 91.
Фиг. 90.
38
3
Рис. 92.
Рис. 93.
356. Имеет ли такое же давление давление на стол кирпичей, расположенный, как показано на Рис. 92?
357. Два кирпича подводятся друг к другу, как показано на рисунке 93. Действуют ли одни и те же силы на опору и давление в обоих случаях?
358. Розетки запрессовываются из специальной массы (Барка-Литовит), действующей на нее силой 37 ° С.5 кН. Квадратный выход 0,0075 м2. Какое давление в розетке?
359. Площадь дна поддона 1300 см2. Подсчитайте, какое давление поддон на столе, если в него наливается вода объемом 3,9 литра.
360. Какое давление на пол производит мальчик, масса которого составляет 48 кг, а площадь подошвы его обуви 320 см2?
361. Спортсмен, масса которого 78 кг, стоит на лыжах. Длина каждой лыжи 1,95 м, ширина 8 см. Какое давление у спортсмена на снегу?
362.Токарный станок массой 300 кг опирается на фундамент на четырех опорах. Определите давление машины на фундамент, если площадь каждой ноги составляет 50 см2.
363. Лед выдерживает давление 90 кПа. Будет ли на этом льду трактор массой 5,4 тонны, если он будет опираться на гусеницы общей площадью 1,5 м2?
364. Двухосный прицеп с грузом имеет массу 2,5 тонны. Определите давление, оказываемое прицепом на дорогу, если площадь контакта каждого колеса с дорогой составляет 125 см2.
365. На железнодорожной двухосной платформе поставлено артиллерийское орудие массой 5,5 тонны. Насколько увеличилось давление платформы на рельсы, если площадь контакта колеса с рельсом 5 см2?
366. Рассчитать давление, оказываемое на рельсы четырехосной нагруженной каретки массой 32 тонны, если площадь соприкосновения колеса с рельсом 4 см2.
39
Рис. 95.
Рис. 96.
367. Какое давление оказывает гранитный столб на грунт, объем которого составляет 6 м3, если площадь основания равна 1.5 м *?
368. Можно ли гвоздем поставить давление 105 кПа? Посчитайте, какое усилие для этого нужно приложить к шляпке ногтя, если площадь ногтя 0,1 мм2.

400. Почему на Холлидице тротуар посыпался песком?
Для увеличения коэффициента трения. В этом случае вероятность поскользнуться и упасть будет меньше.

401. Почему зимой задние колеса некоторых грузовиков обвязываются цепями?
Для того, чтобы увеличить коэффициент трения и тем самым практически предотвратить скольжение между колесами автомобиля и мусором на дорожном полотне.

402. Для чего, когда вы спускаетесь, одно колесо тележки иногда фиксируется так, чтобы оно не вращалось?
Для увеличения трения между тележкой и дорогой. В этом случае скорость клерка будет не очень большой, но безопасной для спуска.

403. Почему на шинах автомобилей колесных тракторов делают глубокий рельефный рисунок (протектор)?
Для увеличения коэффициента трения между колесами дорого. В этом случае сцепление с землей будет более эффективным.

404. Почему при падении трамвайных линий, проходящих возле парков, бульваров и скверов, вывешивается предупреждающий знак «Осторожно, листопад!»?
Сухие листья уменьшают сцепление колесного трамвая с рельсами, в результате чего может произойти пробуксовка колес, тормозной путь трамвая также увеличится.

405. Почему после дождя грунтовая дорога скользкая?
Вода на поверхности Земли является смазкой и поэтому снижает коэффициент трения.

406. Почему после дождя опасно ехать на автомобиле по грунтовой дороге под уклоном?
Потому что вода на поверхности дороги снижает коэффициент трения.

407. Почему некоторые мастера смазывают мыльный винт перед тем, как вкрутить его в скрепляемые детали?
Мыло служит смазкой и снижает коэффициент трения. В этом случае процесс закручивания шурупа будет проще.

408. Почему штапель, за который спускается корабль в воду, обильно смазан?
Для уменьшения коэффициента трения между спускаемым судном и скобами и тем самым облегчения процесса спуска.

409. Какая выемка на ножке для ногтей?
Для увеличения коэффициента трения. В этом случае молоток будет меньше забиваться грудой гвоздей.

410. Назовите одну или две части велосипеда, изготовленные с увеличением силы скольжения скольжения.
Шина резиновая, колодки тормозные.

411. Какая сила трения возникает при движении карандаша в случаях, указанных на рис. 93, и, b? Где сила трения, действующая на карандаш, относительно оси карандаша в обоих случаях?
а) Сила трения скольжения; Он направлен вдоль оси карандаша в сторону, противоположную его движению,
б) сила трения качения; Он направлен перпендикулярно оси карандаша в сторону, противоположную его движению.

412. Грузовик движется (рис. 94). Какого рода трение возникает между: а) столом и колесами; б) груз и тележка; в) оси колес и корпус тележки?
а) сила трения качения;
b) силу трения мира, если груз опирается на тележку, или силу трения скольжения, если груз движется;
c) Сила трения скольжения.

413. Почему кирпичи не лепить (рис. 95 и 96)? Какая сила удерживает их в покое? Изобразите силы, действующие на кирпичи.

414.Полоса перемещается вправо (рис. 97). Где сила трения направлена ​​в сторону стержня; По поводу поверхности, по которой движется штанга?
Что касается штанги, сила трения скольжения направлена ​​влево (против направления движения). Что касается поверхности, по которой движется штанга, сила трения направлена ​​вправо (по направлению движения).

415. Лестница у стены занимает положение, показанное на рисунке 98. Укажите направление силы трения в местах соприкосновения лестницы со стеной и полом.

416. Полоса движется равномерно (рис. 99). Где направление: а) сила упругости горизонтальной части нити; б) вертикальная часть резьбы; в) Сила трения скольжения, действующая на поверхность стола, на стержень? Что равносильно помощи этих сил?

417. Колесо автомобиля падает (рис. 100). Если сила удара направлена ​​между опорным колесом и дорогой, которая действует: а) на колесо; б) в дороге? Где сила упругости дороги?

418.Книга прижимается к вертикальной поверхности (рис. 101). Изобразите графически направление гравитации и трения мира, действующих на книгу.

419. Тележка равномерно движется вправо (см. Рис. 94). Какая сила приводит к перемещению поставляемого к нему груза? Что это за сила для равномерного движения?
Груз, лежащий на тележке, приводится в движение силой трения упора, направленной вправо. При равномерном движении техника эта сила равна нулю.

420.Конвейер равномерно перемещает ящик с грузом (без проскальзывания). Когда грубая сила трения направлена ​​между конвейерной лентой и выдвижным ящиком, когда ящик: а) поднимается; б) перемещение по горизонтали; в) пропущено?
а) вверх по конвейеру; б) он равен нулю; в) вверх по конвейеру.

421. Равна ли сила тяги силе трения, если автобус движется без равномерного скольжения: 1) по горизонтальной траектории; 2) Вверх по наклонному участку дорожки?
Если автобус движется равномерно по горизонтальному участку пути, то удерживающая сила равна силе тяги за вычетом силы сопротивления воздуха.

422. Парашютист, масса которого 70 кг, спускается равномерно. Какая сила сопротивления воздуха действует на парашютиста?

423. На динамометре штанга перемещается равномерно (см. Рис. 97). Какая сила удара между грифом и поверхностью стола? (Цена деления динамометра 1 н.)
При равномерном движении штанги сила трения скольжения между штангой и поверхностью стола равна силе упругости пружины динамометра.Следовательно, в этом случае динамометр показывает нам значение силы трения скольжения. Согласно рис. 97 Равно 4N.

424. Пилы пил разводятся в разные стороны от плоскости пилы. На рис. 102 показано движение пилы в неразбавленном и разбавленном виде. Какой пилой труднее резать? Почему?
Непроявленную пилу резать сложнее, так как в этом случае боковые поверхности пилы плотно прилегают к дереву и между ними возникает большая сила трения.

425. Приведите примеры, когда трение приносит пользу, а когда вред.
Повышение трения при ходьбе, беге, движении на транспорте, перемещении товаров по конвейеру. Трение наносит вред трущимся деталям различных механизмов, где стирать поверхности нежелательно.

426. На уроке физкультуры мальчик равномерно скатывается по веревке. Под действием какой силы находится это движение?
Под действием силы тяжести и силы трения скольжение.

427. Корабль навстречу трем баржам, последовательно соединенным друг с другом. Мощность водонепроницаемости первой баржи 9000 Н, второй 7000 Н, третьей 6000 Н. водонепроницаемость самого судна 11 кН. Определите силу тяги, развиваемой судном при буксировке этих барж, считая, что баржи движутся равномерно.

428. На движущемся по горизонтали автомобиле сила тяги двигателя составляет 1,25 кН, сила трения 600 Н и сила сопротивления воздуха 450 Н.Что равно равнодействующей этих сил?

429. Можно однозначно утверждать, что приращение силы сопротивления ЗВ составляет 3 мн, если скорость тела, движущегося в некоторой среде с коэффициентом сопротивления 0,01, увеличится на 0,3 м / с?
Однозначно так сказать нельзя, так как мощность сопротивления в вязких средах устанавливается специально. На малых скоростях движения он пропорционален скорости, на больших — квадрату.

430. Троллейбус тронут с места и за 30 с набирает импульс 15 104 кг-м / с.Определите силу сопротивления движению, если сила тяги, развиваемая троллейбусом, составляет 15 кН.

431. Автомобиль массой 103 кг при движении испытывает силу сопротивления, равную 10% от его веса. Какой должна быть сила тяги, развиваемая автомобилем, чтобы двигаться с постоянным ускорением 2 м / с2?

434. Велосипедист, мчавшийся со скоростью 11 м / с, резко сбавил скорость. Коэффициент шлифования скользящих шин по сухому асфальту составляет 0,7. Определить ускорение велосипедиста при торможении; время торможения; Тормозной путь велосипедиста.

435. Какую силу необходимо приложить в горизонтальном направлении к автомобилю массой 16 т, чтобы снизить его скорость на 0,6 м / с за 10 с; На 1 с? Коэффициент трения 0,05.

436. На какой скорости мотоциклист сможет проехать по горизонтальной плоскости, описывая дугу радиусом 83 м, если коэффициент трения грунтовой резины равен 0,4?

Все механики с юности имеют запоминающуюся картинку со схемой движения автомобиля по кривой, когда его внешние колеса проходят больший путь, чем внутренние.С его помощью во многих учебниках для водителей объясняется назначение и принцип действия дифференциала. Часто все сводится к тому, что дифференциал позволяет ведущим колесам вращаться с разной скоростью и, таким образом, обеспечивает нормальное движение автомобиля на поворотах.

Подобные пояснения не совсем некорректны, но суть дифференциала слишком упрощена и суть операции не раскрывается. Конечно, в серьезных книгах все сказано правильно.Говорят, что предназначение межкрементального дифференциала на автомобиле состоит в том, чтобы распределять крутящий момент строго одинаково между ведущими колесами одного моста, а межостатический дифференциал — в распределении крутящего момента между ведущими мостами, одинаково или в равной степени. оптимальная пропорция (несимметричный дифференциал).

«Дифференциал — это механизм, ведущие колеса которого вращаются независимо от друг друга».

Строго говоря, вращаются «в зависимости», но ладно, — сказано что-то похожее на правду, а остальное не словом, чтобы не беспокоить людей без специальной подготовки.

Зеленин С.Ф., Молоков В.А. Учебник по устройству автомобиля, М., «РоссияАвокнига», 2000, 80 с. Тираж 15000 экз.

Цитата из этой книги:

« Дифференциал предназначен Для распределения крутящего момента между осями ведущих колес при вращении автомобиля и при движении по неровностям дороги. Дифференциал позволяет колесам вращаться с разной угловой скоростью и проходить неравномерный путь без проскальзывания относительно дорожного покрытия.

Другими словами, 100% крутящего момента, приходящего на дифференциал, можно распределить между ведущими колесами как 50 x 50, так и в другой пропорции (например, 60 x 40). К сожалению, пропорция может быть 100 х 0. Это означает, что одно из колес стоит на месте (в яме), а другое в это время упадет (для сырой земли, глины, снега).

Что поделаешь! Нет ничего абсолютно правильного и совершенного, но такая конструкция позволяет машине разворачиваться без движения, и водитель не меняет каждый день полностью изношенные шины.

Рис.38 Главная передача с дифференциалом

1 — полуоси; 2 — ведомая шестерня; 3 — ведущая шестерня; 4 — шестерня; 5 — Спутники

Это уже не упрощение, а просто введение в заблуждение читателей. Здесь, кроме второго предложения и иллюстрации, все не так (в первое предложение нужно вставить слово «ограбление», а точку поставить после слова «колеса» и т. Д.).

Лишь однажды в учебнике по профессиональному образованию мне довелось встретить правильное и в то же время простое и наглядное объяснение сути работы дифференциала.Это было давно и помню только, что это был учебник для машинистов зерноуборочных комбайнов.

Здесь читателю было предложено представить, что две полуосевые конические шестерни «развернуты» в две зубчатые рейки, эти рейки лежат на воображаемом столе, а сателлит размещен в виде прямой шестерни. Выглядит это так:

Объяснение сущности дифференциальной работы основано на ее конструкции и третьем законе Ньютона, который гласит: сила действия равна модулю и противоположна направлению противодействия.На следующем рисунке показано мощное взаимодействие спутника с рельсами, когда движущая сила d приложена к оси сателлита, и этот сателлит толкает оба рельса на столе, а также сила сопротивления левой и правой палуб со львом. и от прав то же самое (трение о поверхность воображаемого стола. И каждое из них равно половине общего сопротивления S. Сателлитные силы передаются на рельсы в точках зацепления спутника с палубами .Благодаря равенству сопротивлений сопротивления Льву и справа, равным между собой и движущим силам на зубцах сателлита, каждый из которых равен половине движущей силы D. Поскольку равные силы приложены к двум Зубцы сателлита, которые равны расстояниям от его оси, сателлит находится в равновесии и не вращается. Следовательно, все три детали движутся в одном направлении и с одинаковой скоростью, а именно с той скоростью, с которой движется ось спутника и которая задается двигателем.

Данная ситуация соответствует установленному автомобилю передвижению по дороге с хорошим сцеплением с дорогим.

А теперь представьте, что левый рельс своим движением по столу «наткнулся» на масляное пятно. При этом сила сопротивления его движению (сила трения о стол) уменьшилась, а сила сопротивления движению правого рельса осталась прежней. В какой-то момент равновесие сил на зубах сателлита нарушается: нагрузка на левый зубец становится меньше нагрузки, действующей на его правый зуб.Другими словами, сателлиту стало легче толкать левую рейку, чем правую. Поэтому он начинает вращаться по часовой стрелке, как показано на следующем рисунке.

Благодаря вращению спутника движение правого рельса замедляется, а левого рельса наоборот ускоряется. Затем правый гребень полностью останавливается, и сателлит продолжает вращаться. Его ось продолжает двигаться с той же скоростью, что и раньше, поскольку эта скорость задается двигателем. Но поскольку правая железная дорога стоит, вращающийся спутник проходит через нее.В момент времени, показанный на рисунке, правый зубец сателлита стоит на месте, «упираясь» в зубец неподвижного рельса. Но наоборот, левый зуб спутника движется вдвое быстрее, чем ось самого спутника. Все это соответствует ситуации, когда одно из ведущих колес медленно движущегося автомобиля едет, например, по обширному пятну льда, а второе остается на сухом покрытии с хорошим сцеплением. То есть машина останавливается, и колесо падает на лед, поворачиваясь в два раза быстрее, чем раньше, когда оба колеса катились с одинаковой скоростью.

Строго говоря, баланс баланса сил на зубцах-сателлитах выше указан неверно и только потому, что, как мне кажется, легче понять происходящее. На самом деле баланс сил всегда сохраняется, только для его рассмотрения все же следует учитывать силы, вызывающие ускорение левой стойки и замедляющие правую. Эти не учтенные нами силы исчезают с момента полной остановки правого рельса. В то же время удвоенная скорость левого рельса становится постоянной.И тогда ситуация полностью соответствует следующему рисунку.

Здесь восстановился баланс сил, точнее исчезли динамические силовые составляющие (те, которые вызывали ускорение одного рельса и замедление другого). Правый рельс стоит, спутник вращается, а левый рельс движется равномерно с удвоенной скоростью. Очень важно отметить, что равновесие сил перешло на новый уровень. Теперь равные силы на левом и правом зубах-сателлитах стали значительно меньше прежних.В силу третьего закона Ньютона эти силы не могут превышать движущую силу, которая может быть приложена к рельсу, находящемуся на пятне масла, или к колесу, находящемуся на пятне льда. Другими словами, если одно колесо стоит на сухой дороге, а противоположное будет на льду или в грязи, это не означает, что 100% крутящего момента передается от двигателя на осевое колесо, как указано в книге. упомянутый выше. Этот момент всегда и во всех условиях делится дифференциалом поровну между колесами, но он не может быть больше сцепления одного из колес с дорогой и колеса, у которого сцепление меньше.

Только если в этих условиях заблокировать дифференциал, то есть выключить его с работы, так или иначе, жестко соединив полуоси, можно передать на колесо, стоящее на сухой дороге, подавляющая часть крутящий момент, который может развить двигатель. При этом буксирование прекращается, оба колеса будут вращаться с одинаковой скоростью, но подавляющую часть общей силы тяги будет обеспечивать только одно из этих колес.

Мне кажется, что с помощью модели с зубчатыми рейками можно наглядно объяснить все остальные режимы работы межкрементного дифференциала.Например, иногда возникает ситуация при торможении двигателем. Представьте, что машина движется под откосом по сухой дороге с пятнами льда. Водитель тормозит двигатель. В этом случае движущей силой является сила инерции массы автомобиля. А сила сопротивления движению — это сила, приложенная к осям сателлитов дифференциала со стороны двигателя. Одно из колес движется по ледяному пятну. Сила сцепления этого колеса с дорогой резко уменьшается, и оно начинает вращаться в обратном направлении.Здесь происходит то же самое, что и с рельсами, если ось сателлита фиксируется, но оставляет ему свободу вращения вокруг этой оси, то есть имитирует ситуацию, когда ось сателлита тормозится или поддерживается двигателем. Если вы переместите одну из шестерен вперед, то сателлит начнет вращаться, и второй рельс отойдет назад. Здесь рельс, движущийся вперед, соответствует колесу по сухой дороге, а движущийся назад грабли — колесу по льду, вращающемуся в противоположном направлении.На мой взгляд, вращение компенсирующего колеса в противоположную сторону очень наглядно демонстрирует «желание» дифференциала выполнять свое предназначение и выравнивать силы на двух колесах ведущего моста. В данном случае это силы торможения. Благодаря их уровню вероятность заноса автомобиля исключена или сильно снижена.

Можно рассмотреть еще много ситуаций, возникающих при срабатывании дифференциала. Но полагаю, что этого достаточно, чтобы убедиться: — межколесный дифференциал всегда делит Крутящий момент от двигателя одинаково Между двумя колесами одного ведущего моста.

А теперь вернемся к упомянутой в самом начале картинке с автомобилем, движущимся по повороту. Если у автомобиля задний привод, то два задних колеса, получающие одинаковый крутящий момент, преобразуют этот крутящий момент в две одинаковые силы тяги (если колеса колеса имеют одинаковый диаметр, одинаковое давление насоса и одинаковые части автомобиля). . И две одинаковые тяговые силы стремятся толкнуть машину по прямой. Поэтому водителю при прохождении поворота приходится крепко держать руль.Собственно говоря, дифференциал на такой машине не столько помогает, сколько мешает прохождению поворота. Но это напрямую способствует сопротивлению движению по прямой (вместе с углом установки передних колес).

У переднеприводного автомобиля ситуация несколько иная. Здесь силы тяги на двух колесах тоже одинаковые, но они «вращаются» вместе с вращающимися колесами. Поэтому, например, переднеприводной машине легче выйти из колеи глубокого пробуксовки: повернутые передние ведущие колеса активно тянут там, где это необходимо.А задний привод, задние ведущие колеса активно толкают машину по колее.

Здесь добавлена ​​лишь небольшая часть того, что водители должны были знать о работе дифференциала, и потребовалось много слов и изображений. Так может быть тем, кто ограничивается пресловутой картинкой с разным пробегом разных колес на повороте? Может быть. Но я полагаю, что стоит если не вдаваться в подробные объяснения, то хотя бы просто написать, для чего этот механизм действительно предназначен.А кто хочет дотянуться до сути, найдет, где об этом почитать. И вообще ничего, что способствовало бы вашему неправильному пониманию этой сути.

Красный свет, зеленый свет: силы трения, дороги и шины — Урок

(0 Рейтинги)

Быстрый просмотр

Уровень оценки: 8 (7-9)

Требуемое время: 45 минут

Зависимость урока:

Тематические области: Физические науки, физика

Ожидаемые характеристики NGSS:


Поделиться:

Резюме

Основываясь на своем понимании сил и законов движения Ньютона, студенты узнают о силе трения, особенно в отношении автомобилей.Они исследуют трение между шинами и дорогой, чтобы узнать, как оно влияет на движение автомобилей во время вождения. В сопутствующем упражнении по обучению грамоте учащиеся исследуют тему конфликта в литературе, разницу между внутренним и внешним конфликтом, а также различные типы конфликтов. Истории используются для обсуждения методов управления и разрешения конфликтов и межличностных трений. Эта инженерная программа соответствует научным стандартам нового поколения (NGSS).

Инженерное соединение

Инженеры используют свое понимание силы трения для проектирования безопасных дорог, шин, автомобилей и тормозов.Транспортные и автомобильные инженеры следят за тем, чтобы дороги и шины обеспечивали необходимое трение, потому что трение обеспечивает сцепление и контроль для безопасного вождения, особенно в обледенелых или влажных условиях. Даже проектирование того, как бумага перемещается через копировальный аппарат, требует понимания трения. Инженеры также уменьшают силу трения между движущимися механическими частями (в двигателях, инструментах, протезах и т. Д.), Поэтому детали работают более плавно и служат дольше.

Цели обучения

После этого урока учащиеся должны уметь:

  • Объясните, как трение связано с движением автомобилей (движение вперед, остановка, повороты).
  • Определите трение как силу, которая сопротивляется движению объекта, движущегося относительно другого объекта
  • Предсказать способы улучшить движение автомобилей по снегу и льду и объяснить, почему
  • Связать инженерное дело с силами трения, особенно при конструировании автомобилей.

Образовательные стандарты

Каждый урок или мероприятие TeachEngineering соотносится с одним или несколькими научными предметами K-12, образовательные стандарты в области технологий, инженерии или математики (STEM).

Все 100000+ стандартов K-12 STEM, охватываемых TeachEngineering , собираются, обслуживаются и упаковываются сетью стандартов достижений (ASN) , проект D2L (www.achievementstandards.org).

В ASN стандарты иерархически структурированы: сначала по источникам; например , по штатам; внутри источника по типу; например , естественные науки или математика; внутри типа по подтипу, затем по классу, и т. д. .

NGSS: научные стандарты нового поколения — наука
Ожидаемые характеристики NGSS

МС-ПС2-2. Запланируйте расследование, чтобы получить доказательства того, что изменение движения объекта зависит от суммы сил, действующих на объект, и массы объекта.(6-8 классы)

Вы согласны с таким раскладом? Спасибо за ваш отзыв!

Нажмите, чтобы просмотреть другие учебные программы, соответствующие этим ожиданиям от результатов.
Этот урок посвящен следующим аспектам трехмерного обучения NGSS:
Наука и инженерная практика Основные дисциплинарные идеи Пересекающиеся концепции
Планируйте расследование индивидуально и совместно, а также при разработке: определите независимые и зависимые переменные и элементы управления, какие инструменты необходимы для сбора данных, как будут регистрироваться измерения и сколько данных необходимо для подтверждения претензии.

Соглашение о выравнивании: Спасибо за ваш отзыв!

Научные знания основаны на логических и концептуальных связях между доказательствами и объяснениями.

Соглашение о выравнивании: Спасибо за ваш отзыв!

Движение объекта определяется суммой действующих на него сил; если общая сила, действующая на объект, не равна нулю, его движение изменится. Чем больше масса объекта, тем больше сила, необходимая для достижения такого же изменения движения.Для любого данного объекта большая сила вызывает большее изменение в движении.

Соглашение о выравнивании: Спасибо за ваш отзыв!

Все положения объектов и направления сил и движений должны быть описаны в произвольно выбранной системе отсчета и произвольно выбранных единицах размера. Чтобы делиться информацией с другими людьми, необходимо также поделиться этим выбором.

Соглашение о выравнивании: Спасибо за ваш отзыв!

Объяснения стабильности и изменений в естественных или спроектированных системах могут быть построены путем изучения изменений во времени и сил в различных масштабах.

Соглашение о выравнивании: Спасибо за ваш отзыв!

Международная ассоциация преподавателей технологий и инженерии — Технология
ГОСТ Предложите выравнивание, не указанное выше

Какое альтернативное выравнивание вы предлагаете для этого контента?

Больше подобной программы

Предварительные знания

Силы, законы движения Ньютона

Введение / Мотивация

Вспомните, когда вы в последний раз ездили на автобусе или машине.Почему на некоторых машинах колеса крутятся на зеленый свет, а на других машинах едет вперед, не крутясь? Почему машины с визгом останавливаются, когда водитель нажимает на тормоза? Что заставляет машину, идущую со скоростью 60 миль в час, оставаться на шоссе, безопасно двигаясь вперед? Ответ на все эти вопросы — сила ТРЕНИЯ (и, конечно, законы движения Ньютона)! Инженеры должны понимать силу трения, чтобы они могли проектировать безопасные дороги, шины, автомобили и тормоза. Инженеры также стараются уменьшить силу трения между движущимися механическими частями, чтобы они прослужили дольше.

Когда шины автомобиля начинают вращаться, трение между дорогой и шинами заставляет автомобиль двигаться вперед. Точно так же, когда автомобиль поворачивает за угол, трение между дорогой и шинами не дает автомобилю соскользнуть с обочины дороги (помните первый закон Ньютона? прямая линия — прямо с дороги!) Когда автомобиль замедляется, трение между дорогой и шинами помогает остановить машину, так как колеса замедляются.Торможение колес вызывает трение между колесами и тормозными колодками. Очевидно, что трение — очень важная сила, когда вы едете в машине!

Представьте, что трение между дорогой и шинами стало намного меньше. Представьте, что трение между ботинком и льдом такое же, как если бы вы шли по покрытому льдом озеру. Что случилось бы? Машины будут скользить по всей дороге, потому что трение между дорогой и шиной будет недостаточным, чтобы шина «прилипла» к дороге.Автомобилям будет сложно тронуться с места, потому что у них будут крутиться колеса. Также будет сложно останавливаться машинам. Представьте, что вы пытаетесь остановить автомобиль, нажав на педаль тормоза, если бы между шинами и дорогой не было трения — что бы произошло? Даже если бы колеса перестали вращаться, машина продолжала бы скользить по дорогам с низким коэффициентом трения! Автомобиль выскользнул и вылетел из-под контроля, выехав за желтую линию или в канаву!

Иногда, когда идет дождь или снег, трение между шинами и дорогой может быть значительно уменьшено, и возникает опасность скольжения автомобиля.Из-за этой опасности инженеры разработали особо безопасные тормоза — антиблокировочную тормозную систему (ABS) с компьютерным управлением. Поскольку эти тормоза с АБС управляются компьютером, они всегда могут определять, что делает каждое колесо. Поскольку компьютер знает, что делают все четыре колеса, если одно колесо вот-вот начнет буксовать при попытке затормозить, система ABS может отрегулировать три других колеса так, чтобы автомобиль не скользил. Обратитесь к увлекательной и практической деятельности «Гонщики на воздушной подушке»! для студентов, чтобы исследовать трение между двумя поверхностями.Последующие действия в связи с соответствующей деятельностью Насколько далеко? Измерение трения с использованием различных материалов для учащихся, чтобы изучить, как разные текстуры обеспечивают различное трение движущимся по ним объектам.

Однако наша воображаемая дорога с низким коэффициентом трения — не все плохие новости. Если бы автомобиль мог ехать с очень небольшим трением между шинами и дорогой, он был бы намного более экономичным, потому что двигателю не приходилось бы очень много работать, чтобы поддерживать автомобиль в движении. Транспортные и автомобильные инженеры следят за тем, чтобы дороги и шины имели правильное количество трения — слишком большое, и автомобильным двигателям придется выполнять больше работы, чтобы продолжать работать; маловато, и авто бы не прилипало к дороге!

Инженеры используют эту связь между трением и динамикой шины / дороги, чтобы строить дороги из материала, который способствует трению в целях безопасности, но не слишком сильному, чтобы машинам не приходилось слишком много работать, чтобы двигаться вперед.В районах, где много снега, некоторые люди используют специальные шины с металлическими шипами, потому что металлические шипы входят в снег и лед, создавая большее трение, чем одна резина. Иногда дороги настолько обледенели, что людям даже приходится надевать цепи на свои шины, чтобы создать достаточное трение между автомобильными шинами и дорогой, чтобы они не скользили и не скользили по всей дороге.

После урока обратитесь к сопутствующему упражнению «Чтобы сплести двое», чтобы раскрыть идею трений в личных отношениях с помощью историй.

Предпосылки и концепции урока для учителей

Трение — это сила, возникающая при трении предметов друг о друга. Сила трения между дорогой и шиной — это то, что позволяет шине «оттолкнуться» от дороги, тем самым двигая автомобиль вперед (третий закон Ньютона — действие — это сила трения толкания, реакция — это движение автомобиля вперед). Представьте себе машину, которую опускают на домкрат. Когда шины вообще не касаются земли (и поэтому между шинами и землей нет трения), шины могут вращаться, но автомобиль не движется.В тот момент, когда шины впервые касаются земли, они «цепляются» за землю. Этот «захват» — это сила трения между шиной и землей. Когда шины «цепляются» за землю, автомобиль движется вперед. Иногда, например, когда автомобиль идет по льду, грязи или песку, сила трения не достаточна для того, чтобы шины автомобиля цеплялись за землю, и поэтому автомобилю трудно двигаться вперед (он раскручивается или скользит). Таким образом, даже если трение часто рассматривается как сила, противодействующая движению объекта, движение автомобиля было бы невозможно без трения! Трение может замедлить автомобиль при движении по дороге, но это также сила, которая позволяет машине вообще двигаться вперед.Именно сила трения удерживает шины от скольжения по дороге. Точно так же именно трение заставляет автомобиль останавливаться при включении тормозов. Таким образом, именно сила трения заставляет машину ускоряться вперед, а также замедляться до остановки.

Трение также очень важно при повороте автомобиля. Если трение между дорогой и шинами недостаточно, автомобиль будет съезжать с дороги боком, вместо того, чтобы повернуть за угол. У грунтовых дорог меньше трения, чем у дорог с твердым покрытием, поэтому автомобили иногда скользят по углам на грунтовых дорогах по телевизору или в фильмах.Если автомобили едут очень быстро, задние колеса могут скользить, делая автомобиль «рыбьим хвостом» при повороте.

Сопутствующие мероприятия

  • Гонщики на воздушной подушке! — Во время этого упражнения ученики исследуют трение между двумя поверхностями. Студенты строят судно на воздушной подушке с воздушным шаром и компакт-диском, узнав, что воздушный слой под объектом значительно снижает трение, когда он скользит по поверхности.

    Посмотреть это занятие на YouTube

  • Как далеко? Измерение трения с использованием различных материалов — это упражнение показывает, как разные текстуры обеспечивают различное трение движущимся по ним объектам. Студенты экспериментируют с наждачной бумагой и вощеной бумагой (и любым другим материалом, который, по вашему мнению, может быть интересен), чтобы определить, какой материал обеспечивает наименьшее / наибольшее трение.

    Посмотреть это занятие на YouTube

  • Чтобы спутались двое — учащиеся исследуют тему конфликта в литературе. Они узнают разницу между внутренним и внешним конфликтом и различными типами конфликтов. Истории используются для обсуждения методов управления и разрешения конфликтов и межличностных трений.

Закрытие урока

Попросите учащихся объяснить, что, по их мнению, такое трение. Почему он замедляет движение, но в то же время делает его возможным? Почему трение так важно для инженеров, которые строят автомобили? Какие дополнительные применения трения в повседневной жизни и для инженеров? Какой была бы жизнь без трения?

Словарь / Определения

Сила: что-то, что действует извне, чтобы толкать или тянуть объект.Например, взрослый, тащащий ребенка в повозке, оказывает на повозку силу.

Трение: сила, возникающая при трении предметов друг о друга.

Третий закон движения Ньютона: на каждое действие существует равное и противоположное противодействие.

Оценка

Оценка перед уроком

Вопрос для обсуждения: Запрашивайте, объединяйте и обобщайте ответы студентов.

  • Какие силы заставляют машину двигаться со скоростью 60 миль в час, оставаться на шоссе и безопасно двигаться вперед? Почему автомобиль не замедляется, не поворачивает направо и налево или не съезжает с дороги? (Ответ: ТРЕНИЕ и третий закон движения Ньютона. Трение между дорогой и шинами автомобиля заставляет его двигаться в прямом направлении.).

Оценка после введения

Вопрос / ответ: Задайте ученикам и обсудите в классе:

  • Представьте, что вы пытаетесь остановить автомобиль, нажав на педаль тормоза, если бы между колесами и дорогой не было трения.Что случилось бы? (Ответ: Если бы не было трения, колеса остановились бы, но автомобиль продолжал бы скользить и вообще не замедлялся бы. Автомобиль останавливался бы только при столкновении с чем-либо.)
  • Можете ли вы представить себе ситуацию, в которой инженеру может потребоваться меньше трений? Когда инженеру может понадобиться больше трения? (Возможные ответы: инженеры стараются уменьшить трение между движущимися частями, чтобы они не изнашивались так быстро. Увеличение трения — полезный способ замедлить работу — именно так работают тормоза в автомобиле, велосипеде или скутере! есть много других возможных ответов на этот вопрос.)

Итоги урока Оценка

Круглый стол: Разделите класс на группы по 3–5 студентов в каждой. Попросите учащихся каждой команды составить список того, каким образом трение влияет на автомобили, и каждый человек по очереди записывает свои идеи. Студенты разносят список по группе, пока не будут исчерпаны все идеи. Попросите команды прочитать ответы вслух и записать их на доске. (Предложение: трение возникает не только между колесами и дорогой, но и между всеми движущимися частями двигателя, тормозами и т. Д.)

Friction Boggle !: Повторите то же действие, что и выше, за исключением случаев, когда команды зачитывают свои ответы вслух и записывают их на доске, спросите, придумали ли другие команды такая же идея. Если у других команд есть такой же ответ на своих листах, все команды должны вычеркнуть этот ответ в своем списке. Побеждает команда, у которой есть самые «уникальные» идеи!

Мероприятия по продлению урока

Попросите учеников пойти домой и покататься на велосипеде, а затем вернуться в школу с некоторыми наблюдениями о движении велосипеда и трении, которое они создают во время езды на велосипеде.Чем дорожные велосипеды отличаются от горных? Почему шины разные?

Попросите студентов исследовать суда на воздушной подушке. Как они работают? Как быстро они могут идти? По каким поверхностям они могут переходить? Какую роль играет трение в движении судов на воздушной подушке?

использованная литература

Гиттевитт, Пол. Концептуальная физика. Менло-Парк, Калифорния: Аддисон-Уэсли, 1992.

Хаузер, Джилл Франкель.Вещи и гаджеты: создание научных хитростей, которые работают (и зная, почему). Шарлотта, VT: Williamson Publishing, 1999.

Каган, Спенсер. Совместное обучение. Capistrano, CA: Kagan Cooperative Learning, 1994. (Источник для оценки за круглым столом.)

ВанКлив, Дженис. Физика для каждого ребенка: 101 простой эксперимент с движением, теплом, светом, машинами и звуком. Нью-Йорк, штат Нью-Йорк: John Wiley and Sons Inc., 1991.

Вольфсон, Ричард и Джей М.Пасачёв. Физика: для ученых и инженеров. Ридинг, Массачусетс: Addison-Wesley Longman Inc., 1999.

авторское право

© 2004 Регенты Университета Колорадо.

Авторы

Сабер Дурен; Бен Хевнер; Малинда Шефер Зарске; Дениз Карлсон

Программа поддержки

Комплексная программа преподавания и обучения, Инженерный колледж, Университет Колорадо в Боулдере

Благодарности

Содержание этой учебной программы по цифровой библиотеке было разработано за счет гранта Фонда улучшения послесреднего образования (FIPSE), U.S. Министерство образования и Национальный научный фонд ГК-12, грант No. 0338326. Однако это содержание не обязательно отражает политику Министерства образования или Национального научного фонда, и вам не следует предполагать, что оно одобрено федеральным правительством.

Последнее изменение: 13 октября 2021 г.

Принудительное воздействие на пассажиров

Эта основная идея исследуется через:

Противопоставление студенческих и научных взглядов

Ежедневный опыт учеников

На этом уровне концептуального развития многие ученики средних классов по-прежнему будут придерживаться твердых взглядов, основанных на их повседневном опыте.

Эти виды рассматриваются в фокусе идей Толкает и тянет и Что такое сила?

Студенты иногда не могут правильно определить силы, которые действуют на них, когда они путешествуют в качестве пассажира в поезде, автобусе или автомобиле. Эта путаница часто возникает из-за взгляда на движение транспортного средства изнутри.

Когда машина внезапно тормозит, учащиеся считают, что их выбросило вперед внезапно. « вперед» сила.

Исследования: Mitchell (2007)

Другой повседневный опыт — это кажущаяся боковая сила, которая внезапно действует на пассажиров и предметы в машине, когда машина поворачивает в крутом повороте.

Научная точка зрения

Когда автомобиль тормозит, сила тормозов замедляет автомобиль, но не ограничивает пассажиров — они продолжают двигаться вперед со скоростью, на которой автомобиль двигался непосредственно перед торможением. Обитателю кажется, что их выбросило вперед.Фактически, они продолжают двигаться без каких-либо новых сил, действующих на них, пока не столкнутся с силой удерживающего ремня безопасности или подушки безопасности.

Ремни безопасности и подушки безопасности предназначены для приложения тормозных сил к пассажирам в течение длительного периода времени, поэтому сила удара, воздействующая на пассажиров, будет уменьшена, а шансы на выживание пассажиров увеличатся.

Точно так же, когда автомобиль поворачивает на крутом повороте, ремень безопасности и автокресло должны давить на пассажиров, чтобы они изменили направление своего движения и стали такими же, как у поворачивающего автомобиля.

Критические обучающие идеи

  • Когда автомобиль тормозит, он замедляется, но не сдерживаемый пассажир или какой-либо предмет в машине.
  • Не сдерживаемый пассажир или объект будет остановлен силой, оказываемой на него другим объектом, например, ветровым стеклом, рулевым колесом или подушкой безопасности.
  • Большая сила замедлит объект быстрее.
  • Подушка безопасности или ремень безопасности предназначены для увеличения времени, необходимого пассажиру для замедления во время торможения, что снижает нагрузку на пассажира.

Исследования: Митчелл (2007), Лафран, Берри и Малхолл (2006)

Изучите взаимосвязь между идеями о силах и движении в Карты развития концепций — Законы движения

Чтобы получить представление об альтернативных концепциях учащихся, прежде чем приступить к разработке следующих, ознакомьтесь с основной идеей Толкает и тянет.

Следует выбирать такие виды деятельности, которые способствуют обсуждению сил, действующих на объекты, которые движутся и неподвижны в повседневной жизни.Основная цель этих обсуждений — определить, находятся ли силы в равновесии друг с другом, или они вызывают толчок или притяжение, которые изменяют скорость и / или направление объекта.

Учащимся необходимо прийти к пониманию того, что все силы уравновешены на неподвижном объекте и что любой дисбаланс приведет к тому, что объект будет либо ускоряться, либо замедляться в направлении дисбаланса.

Более сложное понимание состоит в том, что все силы также находятся в равновесии для любого объекта, движущегося в постоянном направлении с постоянной скоростью.Например, ученикам легче принять идею о том, что силы, действующие на ученика, катающегося на роликовых коньках, уравновешены, если ученик стоит на месте, и им труднее принять эту идею, если ученик катается на роликовых коньках с постоянной скоростью 12 км / ч по прямой. дорожка.

Обучающие задания

Начать обсуждение через общий опыт

Прикрепите резиновую ленту или отрезок резинки к лабораторной тележке со свободно вращающимися колесами. Попросите учащихся предсказать, что произойдет, если они потянут тележку с помощью резинки или резинки, чтобы она оставалась растянутой на фиксированную длину.Попросите учащихся выполнить задание и объяснить свои наблюдения с точки зрения сбалансированных и неуравновешенных сил. Почему трудно поддерживать тягу тележки с использованием этого метода только в течение короткого периода времени?

Исследования: Gunstone & Mitchell (1998), Loug hran, Berry & Mulhall (2006)

Бросьте вызов существующим идеям

POE (Прогноз-Наблюдение-Объяснение): используя ту же тележку, что и выше, используйте пластилин, чтобы свободно прикрепить пластиковую фигурку к тележке. Попросите учащихся предсказать, что произойдет, если тележка на высокой скорости столкнется с неподвижным кирпичом или другой тележкой.Студенты должны объяснить, что они наблюдают, с точки зрения сбалансированных и несбалансированных сил. Обсудите кажущуюся «прямую» силу, как видно на фигурке, в терминах общего движения тележки и различных сил, действующих на тележку и фигурку.

Предоставьте открытую проблему для изучения в игре

Поместите круглый стеклянный шарик на плоскую столешницу и предложите ученику заставить его двигаться, дуя на него трубочкой для питья. Попросив учащихся определить задействованные силы, посмотрите, смогут ли они затем изменить его направление, когда оно снова начнет двигаться, просто продув трубочку для питья.Обсудите, как это лучше всего сделать.

Сосредоточьте внимание учащихся на недооцененной детали

В школах есть много отличных ресурсов по безопасности дорожного движения, которые можно использовать для исследования уравновешенных и неуравновешенных сил, действующих при движении и столкновениях транспортных средств. Определите повседневный опыт учащихся с велосипедами, скейтбордами, роликовыми коньками, роликовыми коньками и самокатами. Студенты часто являются экспертами в использовании этого оборудования для выполнения сложных трюков, но им не хватает точного языка, необходимого для определения сбалансированных и неуравновешенных сил и того, где они действуют.

Веб-сайт класса физики

Инерция и правый поворот

Многие студенты-физики не верят в центростремительные («внутренние») силы. Даже после завершения тщательного курса физики такие студенты все равно будут ошибочно полагать, что объект, движущийся по кругу, испытывает внешнюю силу. Возможно, причина, по которой они придерживаются этого заблуждения, связана с их опытом вождения в качестве пассажира в автомобилях и аттракционов.

Представьте, что вы пассажир в машине, которая поворачивает направо. Когда машина начинает поворот направо, вы часто чувствуете, что — это как будто вы скользите влево. Автомобиль поворачивает вправо из-за внутренней силы , но вы, , чувствуете , как будто вас толкают влево или наружу. На самом деле машина начинает поворот (вправо), а вы продолжаете движение по прямой. Это движение можно лучше понять, изучив приведенную ниже анимацию.

Обратите внимание на анимацию, что пассажир (отмечен синим цветом) продолжает движение по прямой в течение короткого периода времени после того, как автомобиль начинает поворот. Фактически, пассажир следует по прямой, пока не ударится по плечу водителя (выделено красным). После удара по водителю к пассажиру прилагается сила, заставляющая его повернуть вправо и, таким образом, завершить поворот.

Для поворота по кругу требуется направленная внутрь сила.Это требование направленной внутрь чистой силы известно как требование центростремительной силы. В отсутствие какой-либо чистой силы движущийся объект (например, пассажир) продолжает движение по прямой с постоянной скоростью. Это первый закон движения Ньютона. Пока машина начинает разворачиваться, пассажир и сиденье начинают с угла вправо. В каком-то смысле машина начинает выскальзывать из-под пассажира. После удара водителя пассажир теперь может повернуться вместе с автомобилем и испытать круговое движение.На пассажира никогда не действует какая-либо внешняя сила. Пассажир движется либо прямо в отсутствие силы, либо движется по круговой траектории при наличии силы, направленной внутрь.


Для получения дополнительной информации о физических описаниях движения посетите The Physics Classroom Tutorial. Доступна подробная информация по следующим темам:

Ускорение

Чистая сила и ускорение

Круговое движение и тангенциальная скорость

Круговое движение и ускорение

Требование центростремительной силы

Диаграммы свободного тела

Сумма сил

Первый закон Ньютона — Силы, ускорение и законы Ньютона — AQA — Объединенная научная редакция GCSE — Трилогия AQA

Примеры объектов с неравномерным движением

Первый закон Ньютона также может быть использован для объяснения движения объектов, перемещающихся с не- равномерное движение.Сюда входят ситуации, когда меняются скорость, направление или и то, и другое. Например, когда автомобиль ускоряется, движущая сила двигателя больше, чем силы сопротивления. Результирующая сила не равна нулю.

Другие примеры:

  • в начале бега бегун испытывает меньшее сопротивление воздуха, чем его тяга, поэтому он ускоряется
  • объект, который начинает падать, испытывает меньшее сопротивление воздуха, чем его вес, поэтому он ускоряется
Если силы, действующие на объект, не уравновешены, результирующая сила не равна нулю.

Силы на подводной лодке

Подводная лодка выше имеет как вертикальные силы, так и горизонтальные силы, действующие на нее.Горизонтальные силы не влияют на его вертикальное движение, а вертикальные силы не влияют на его горизонтальное движение.

Горизонтальные силы равны по величине и противоположны по направлению. Они уравновешены, поэтому результирующая горизонтальная сила равна нулю. Это означает, что нет горизонтального ускорения. Вертикальные силы равны по величине и противоположны по направлению. Они уравновешены, поэтому результирующая вертикальная сила также равна нулю. Это означает отсутствие результирующего вертикального ускорения.

Подводная лодка продолжит движение, оставаясь неподвижным или двигаясь с постоянной скоростью. Если подводная лодка движется, невозможно определить, в каком направлении она движется, только по силам, только то, что она будет продолжать движение в том же направлении с той же скоростью.

Автор: alexxlab

Добавить комментарий

Ваш адрес email не будет опубликован. Обязательные поля помечены *